You are on page 1of 77

Q.1) Explain briefly features of an IDEAL management control system?

Management control is a process of assuming that resources are obtained and used effectively and efficiently in the
accomplishment of the organizations objectives. It is a fundamental necessity for the success of a business and hence
from time to time the current performance of the various operations is compared to a predetermined standard or ideal
performance and in case of variance remedial measures are adopted to confirm operations to set plan or policy.
Some of the features of MANAGEMENT CONTROL SYSTEM are as follows:

Total System: MANAGEMENT CONTROL SYSTEM is an overall process of the enterprise which aims to fit
together the separate plans for various segments as to assure that each harmonizes with the others and that the
aggregate effect of all of them on the whole enterprise is satisfactory.

Monetary Standard: MANAGEMENT CONTROL SYSTEM is built around a financial structure and all the
resources and outputs are expressed in terms of money. The results of each responsibility centre in respect to
production and resources are expressed in terms of a common denominator of money.

Definite pattern: It follows a definite pattern and time table. The whole operational activity is regular and
rhythmic. It is a continuous process even if the plans are changed in the light of experience or technology.

Coordinated System: It is a fully coordinated and integrated system.

Emphasis: Management control requires emphasis both on the search for planning as well as control. Both
should go hand in hand to achieve the best results.

Function of every manager: Manager at every level as to focus towards future operational and accounting data,
taking into consideration past performance, present trends and anticipated economic and technological changes.
The nature, scope and level of control will be governed by the level of manager exercising it.

Existence of goals and plans: MANAGEMENT CONTROL SYSTEM is not possible without predetermined
goals and plans. These two provide a link between such future anticipations and actual performance.

Forward looking: MANAGEMENT CONTROL SYSTEM is on the basis of evaluation of past performance
that the future plans or guidelines can be laid down. Management Control involves managing the overall activity
of the enterprise for the future. It prevents deviations in operational goals.

Continuous process: It is a continuous process over the human and material resources. It demands vigilance at
every step. Deciding, planning and regulating the activities of people associated in the common task of attaining
the objectives of the organization is a the primary aim of MANAGEMENT CONTROL SYSTEM.

People oriented: It is the managers, engineers and operators which implement the ideas and objectives of the
management. The coordination of the main division of an organization helps in smoother operations and less
friction which results in the achievement of the predetermined objectives.

Scope of control
MANAGEMENT CONTROL SYSTEM is an important process in which accounting information is used to
accomplish the organizations objectives. Therefore the scope of control is very wide which covers a very wide range
of management activities.

Policies control: Success if a business depends on formulation of sound policies and their proper
implementation.

Control over organization: It involves designing and organizing the various departments for the smooth
running of the business. It attempts to remove the causes of such friction and rationalizes the organizational
structure as and when the need arises.

Control over personnel: Anything that the business accomplishes is the result of the action of those people who
work in the organization. It is the people, and not the figures, that get things done.

Control over costs: The cost accountant is responsible to control cost sets, cost standards, labour material and
over heads. He makes comparisons of actual cost data with standard cost. Cost control is a delicate task and is
supplemented by budgetary control systems.

Control over techniques: It involves the use of best methods and techniques so as to eliminate all wastages in
time, energy and material. The task is accomplished by periodic analysis and checking of activities of each
department with a view to avoid an eliminate all non-essential motions, functions and methods.

Control over capital Expenditure: Capital budget is prepared for the whole concern. Every project is evaluated
in terms if the advantage it accrues to the firm. For this purpose capital budgeting, project analysis, study of cost
of capital etc are carried out.

Overall control: A master plan is prepared for overall control and all the departments of the concern are
involved in this procedure.

Q.2

What is the concept


process of computation?

of

free

cash

flow

as

applied

to

organization.

Explain

We define net cash flow as net income plus non cash adjustment which typically means net income plus depreciation
though that cash flows cannot be maintained over time unless depreciated fixed assets are replaced. So management
is not completely free to use its cash flows however it chooses. Therefore we define the term free cash flows.

Free cash flow is the cash flow actually available for distribution to investor after the company has made all the
investment in fixed assets and working capital necessary to sustain ongoing operation. When we studied income
statement in accounting the emphasis was probably on the firms net income, which is accounting profit. However
the value of companys operation is determined by the stream of cash flows that the operations will generate now and
in the future. To be more specific, the value of operation depends on all the future expected free cash flows, defined
as after- tax operating profit minus the amount of new investment in working capital and fixed assets necessary to
sustain the business. Therefore the way for managers to make their companies more valuable is to increase their free
cash flow.

Uses of FCF:
1.
Pay interest to debt holders, keeping in mind that the net cost to the company is the after tax interest expense.
2.
Repay debt holders, that is, pay off some of debt.
3.
Pay dividends to shareholders.

4.
5.

Repurchase stock from shareholders.


Buy marketable securities or other non operating assets.

In practice, most companies combine these five uses in such a way that the net total is equal to FCF. For example, a
company might pay interest and dividends, issue new debts, also sell some of its marketable securities. Some of these
activities are cash outflows (paying interest and dividends) and some are cash inflows (issuing debt and selling
marketable securities), but the net cash flow from these five activities is equal to free cash flows.

Computation of free cash flows:


Eg:
Suppose the company had a 2001 NOPAT of $170.3million and depreciation is only the non cash charge which is
$100million then its operating cash flow in 2001 would be NOPAT plus any non cash adjustment on the statement of
cash flows.
Operating cash flow =NOPAT +depreciation (non cash adjustment)
= $17.03 + $100
= $270.3
Company has $1,455million operating assets, at the end of 2000, but $1,800 at the end of 2001.it made a net
investment in operating assets of
Net investment in operating assets = $18, 00 - $1,455 = $345million
If net fixed assets rose from $870million to $1000million however company reported
So its gross investment in fixed assets would be

$100million of depreciation.

Gross investment = net investment + depreciation


= $130 + $100 = $230million
Company free cash flows in 2001 was
FCF = operating cash flow gross investment in operating assets
= $270.3 - $445
= - $174.7million
An algebraically equivalent equation is
FCF = NOPAT - Net investment in operating assets
= $170.3- $345
= - $174.7million
Even though company had a positive NOPAT, its very high investment in operating assets resulted in a negative free
cash flow. Because free cash flow is what is available for distribution to investor, not only was there nothing for
investors, but investor actually had to provide additional money to keep the business ongoing. A negative current
FCF not necessarily bad provided it is due to the high growth or to support the growth. There is nothing wrong with
profitable growth; even it causes negative free cash flow in the short term
Q.3) What is Balance Scorecard? What is the process of implementation and difficulties in
implementation?

The Balanced Scorecard (BSC) is a performance management tool which began as a concept for measuring whether
the smaller-scale operational activities of a company are aligned with its larger-scale objectives in terms of vision and
strategy.
By focusing not only on financial outcomes but also on the operational, marketing and developmental inputs to these,
the Balanced Scorecard helps provide a more comprehensive view of a business, which in turn helps organizations
act in their best long-term interests.

Organizations were encouraged to measurein addition to financial outputswhat influenced such financial
outputs. For example, process performance, market share / penetration, long term learning and skills development,
and so on.
The underlying rationale is that organizations cannot directly influence financial outcomes, as these are "lag"
measures, and that the use of financial measures alone to inform the strategic control of the firm is unwise.
Organizations should instead also measure those areas where direct management intervention is possible. In so doing,
the early versions of the Balanced Scorecard helped organizations achieve a degree of "balance" in selection of
performance measures. In practice, early Scorecards achieved this balance by encouraging managers to select
measures from three additional categories or perspectives:
"Customer," "Internal Business Processes" and "Learning and
Growth."

The balance scorecard suggests that we view the organization


from four perspectives, and to develop metrics, collect data and
analyze it relative to each of these perspectives:
The learning and growth perspective : To achieve
our vision, how will we sustain our ability to change
and improve?

The business process perspective : To satisfy our


shareholders and customers what business processes must we excel at?

The customer perspective : To achieve our vision, how should we appear to our customer?

The financial perspective : To succeed financially, how should we appear to our shareholders?

Implementing a Balanced Scorecard


We can summarize the implantation of a balanced scorecard in four general steps;
1.
Define strategy.
2.
Define measure of strategy.
3.
Integrate measures into the management system.
4.
Review measures and result frequently.
Each of these steps is iterative, requiring the participation of senior executive and employees throughout the
organization

Define Strategy
The balance scorecard builds a link between strategy and operational action. As a result it is necessary to begin the
process of defining a balanced scorecard by defining the organization goals are explicit and what that targets have
been developed.

Define Measures of Strategy


The next step is to develop measures in support of the articulate strategy. It is imperative that the organization focuses
on a few critical measures at this point; otherwise management will be overloaded with measures. Also, it is important
that the individual measures be linked with each other in a cause effect manner

Integrated Measures into the management system


The balanced scorecard must be integrated with the organization formal and informal structure, its culture, and its
human resources practice. While the balanced Scorecard gives some means for balancing measures, the measures can
still become unbalanced by others system in the organization such as compensation policies that compensate the
manager strictly based on financial performance.


Review Measures and result Frequently
Once the balance scorecard is up and running it must be consistently reviewed by senior management. The
organization should be looking for the following

How do the outcome measures say the organization is doing?


How do the driver measures say the organization is doing?
How has the organizations strategy changed since the last review?
How has the scorecard measures changed?

The most important aspects of these reviews are as follows;

They tell management whether the strategy is being implemented correctly and how successfully the strategy
is working.
They show that management is serious about the importance of these measures.
They maintain alignment of measure to ever changing strategies.

Difficulties in implementing Balanced Scorecard

The following problems unless suitably dealt with, could limit the usefulness of the balanced scorecard approach:

Poor correlation between nonfinancial measures and result.

Fixation on financial result. No mechanism for improvement.

No mechanism for improvement.

Measures overload.

Poor Correlation between Nonfinancial measures and result


Simply put there is no guarantee that future profitably will allow targets achievement in any nonfinancial area. This is
probably the biggest problem with the balanced scorecard because there is an inherent assumption that future
profitability does follow from achieving the scorecard measures, identifying the cause effect relationships among the
different measures is easier said than done.
This will be a problem with any system that is trying to develop proxy measures for future performance. While this
does not mean that the balanced Scorecard should be abandoned it is imp that comp adopting such a system
understand that the links between nonfinancial measures and financial performance are still poorly understood.

Fixation on Financial Results


As previously discussed not only are most senior managers well trained and very adept with financial measures but
they also most keenly feel pressure regarding the financial performance of their comp. Shareholder are vocal and the
board of directors often applies pressure on the stakeholders behalf .this pressure often overwhelms the long term
uncertain payback of the nonfinancial measures.

Non mechanism for Improvement


One of the most overlooked pitfalls of the balanced scorecard is that a company cannot achieve Stretch goals if the
Company has no mechanism for improvement .Unfortunately achieving many of these goals require complete shifts in
the way that business is done yet the company often does not have mechanism to make those shifts . The mechanism
available takes additional resource and requires a changed in the company culture. These changes do not happen
overnight nor do they respond automatically to a new stretch targets. Inertia often works against the company
employees are accustomed to a self limited cycle of setting targets, missing those targets and readjusting the targets to
reflect what was actually achieved. Without a method for making improvement, improvements are unlikely to
consistently happen no matter how good the stretch goal sound.

Measurement overload
How many critical measures can one manager track at one time without losing? Unfortunately there is no right answer
to this question except it is more than 1 and less than 50. It too few then the manager is ignoring measures that are
critical to creating success. If it too many then the manager may risk losing focus and trying to do too many things at
once.

Q.5ABC ltd. (MCS-2008) Numerical


Particulars
ROI
Sales
Investment
EBIT

Division X (Rs.)
28%
100 Lacs
25 lacs
7 Lacs

Division Y (Rs.)
26%
500 lacs
100 Lacs
26 lacs

Analyze and comment upon performances of both the divisions


Solution:
Division X
ROI
Profit

=
=
=

(Profit / investment)* 100


(28/100)*25lacs
7lacs

Profit margin

=
=
=

(Profit/sales)*100
(7/100)*100
7lacs

Turnover of investments

=
=
=

(Sales/investment)*100
(100/25)*100
4 times

=
=
=

(Profit / investment)* 100


(26/100)*100lacs
26lacs

Profit margin

=
=
=

(Profit/sales)*100
(26/500)*100
5.2lacs

Turnover of investments

=
=
=

(Sales/investment)*100
(500/100)*100
5 times

Division Y
ROI
Profit

Profit margin of X is better than profit margin of division Y. Turnover of investment of division Y is better than
Division X.
Hence cost management of Division X is better than Division Y.

SET. 2
Q1. MCS designers apparently disagree whether single measure to evaluate the profit performance and
capital investment performance is preferable or SEPARATE measures for each are preferable
COMMENT ?
ANS. There should be different measures used for evaluating profit performance and capital investment performance
as needed.
The goal of performance measurement systems is to implement strategy. In setting up such systems, senior
management selects measures that best represent the company's strategy. These measures can be seen as current and
future critical success factors; if they are improved, the company has implemented its strategy. The strategy's success

depends on its soundness. A performance measurement system is simply a mechanism that improves the likelihood
the organisation will implement its strategy successfully.

Measuring Profitability

There are two types of profitability measurements used in evaluating a profit center, just as there are in evaluating an
organization as a whole. First, there is a measure of management performance, which focuses on how well the
manager is doing. This measure is used for planning, coordinating, and controlling the profit center's day-to-day
activities and as a device for providing the proper motivation for its manager. Second, there is the measure of
economic performance, which focuses on how well the profit center is doing as an economic entity. The messages
conveyed by these two measures may be quite different from each other. For example, the management performance
report for a branch store may show that the store's manager is doing an excellent job under the circumstances, while
the economic performance report may indicate that because of economic and competitive conditions in its area the
store is a losing proposition and should be closed.
The necessary information for both purposes usually cannot be obtained from a single set of data. Because the
management report is used frequently, while the economic report is prepared only on those occasions when economic
decisions must be made, considerations relating to management performance measurement have first priority in
systems design-that is, the system should be designed to measure management performance routinely, with economic
information being derived from these performance reports as well as from other sources.

Capital Investment Measurement

Most proposals require significant new capital. Techniques for analyzing capital investment proposals attempt to find
either (a) The net present value of the project, that is, the excess of the present value of the estimated cash inflows
over the amount of investment required, or
(b) The internal rate of return implicit in the relationship between inflows and outflows. An important point is that
these techniques are used in only about half the situations in which, conceptually, they are applicable.
There are at least four reasons for not using present value techniques in analyzing all proposals.
1) The proposal may be so obviously attractive that a calculation of its net present value is unnecessary. A
newly developed machine that reduces costs so substantially that it will pay for itself in a year is an
example.
2) The estimates involved in the proposal are so uncertain that making present value calculations is believed to
be not worth the effort-one can't draw a reliable conclusion from unreliable data. This situation is common
when the results are heavily dependent on estimates of sales volume of new products for which no good
market data exist. In these situations, the "payback period" criterion is used frequently.
3) The rationale for the proposal is something other than increased profitability. The present value approach
assumes that the "objective function" is to increase profits, but many proposed investments win approval on
the grounds that they improve employee morale, the company's image, or safety.

4)

There is no feasible alternative to adoption. Environmental laws may require investment in a new

program,as an example. The management control system should provide an orderly way of deciding on proposals that
cannot be analyzed by quantitative techniques. Systems that attempt to rank non-quantifiable projects in order of
profitability won't work. Many projects do not fit into a mechanical ranking scheme.
Q.No. 2. What are the different methods to measure profits of a profit center in organizations? Which
different messages each type of measure is likely to convey to managers?
Ans:

When financial performance in a responsibility center is measured in terms of profit, which is the difference

between the revenues and expenses, the responsibility center is called a profit center.Profit as a measure of
performance is especially useful since it enables senior management to use one comprehensive measure instead of
several measures that often point to different directions.
There are two types of profitability measurements in a profit center, just as there are for the organization as a whole.
There is, first, a measure of management performance, in which the focus is on how well the manager is doing.
This measure is used for planning, coordinating and controlling the day-to-day activities of the profit center. Second,
there is a measure of economic performance, in which the focus is on how well the profit center is doing as an
economic entity. The message given by these two measures may be quite different.

Types of Profitability measures:

In order to evaluate the economic performance of a profit center, one must use net income after allocating all costs.
However, in evaluating the performance of manager, any of five different measures of profitability can be used.
1) Contribution Margin: The logic behind using contribution margin as a measure is that fixed expenses are
not controllable by the manager, and therefore he should focus on maximizing the spread between revenue
and expenses. But the problem with this is that some fixed costs are controllable and all fixed costs are
partially controllable. A focus on the contribution margin tends to direct attention away from this
responsibility.

2) Direct Profit: This measure shows the amount that the profit center contributes to the general overhead and
profit of the corporation. It incorporates all expenses incurred in or directly traced to the profit center,
regardless of whether these items are entirely controllable by the profit center manager. A weakness of this
measure is that it does not recognize the motivational benefit of charging headquarters costs.

3) Controllable Profit: Headquarters expenses are divided into two categories: controllable and noncontrollable. The controllable expenses are controlled by business unit manager. Consequently, if these costs
are included in the management system, the profit will be after the deduction of all expenses that are
influenced by profit center manager.
4) Income before Taxes: In this measure, all corporate overhead is allocated to profit centers. The basis of
allocation reflects the relative amount of expense that is incurred for each profit center. If corporate
overheads are allocated to profit centers, budgeted costs, not actual costs, should be allocated. Then the
performance report will show an identical amount in the budget and actual columns for such overheads.

5) Net Income: Here, companies measure performance of domestic profit centers at the bottom line, the
amount of net income after income tax. There are two arguments 1) Income after tax is constant percentage
of the pretax income, so there is no advantage in incorporating income taxes 2) many decisions that have
impact on income taxes are made at headquarters, and it is believed that profit center manager should not be
judged by the consequences of these decisions.
Q3: Explain special characteristics of professional organizations which impact Management Control. What
are interactive controls?

Special Characteristic of a Professional Organization:


1. Goals
A goal of a manufacturing company is to earn a satisfactory profit specially a satisfaction profit, specially a
satisfactory return on assets its principle assets is the skill of its professional staff which doesnt appear on its balance
sheet .return on assets employed therefore is essential meaningless in such organization .their financial goal is to
provide adequate compensation to the professional.
2. Professionals
Professional organization is labour intensive and the labour is of a special type. Research and development
organization use in setting selling price and for other management purposes .standard cost system ,separation of fixed
and variable cost and analyses of variance were built on the foundation are example of organization whose product are
professional service. Professional tends to give in adequate weight to the financial implication of their decision they
want to do the best job they can regardless of its cost.
Because profession are the organization most important resource some authors have advocated that the value of these
profession should be counted as assets the system that does this is called human resource accounting .in the 1970s
many books and articles were written on this subject but few comp actually such a system and we do not know of any
that one current .the problem of measuring the value of human assets is intractable.
3. Output and input measurement
The output of a profession organisation cannot be measured in physical terms, use in setting selling price and for other
management purposes .standard cost system, seperstion of fixed and variable cost and analyses of variance were built
on the foundation. We can measures the number of patient a physician treats n a day and even classify these visit by
type of complaint but this is by no means equivalent to measuring the amt or quality earned is one measures of output
in some professional organization but these monetary amts at most relate to the quantity of service rendered not to
their quality.
Some profession notably scientist engineer, and professional are reluctant to keep track of how they spend their time
and this complicate the track of measuring performance .this reluctant seems to have its root in tradition usually it can
be overcome if senior management is willing to put appropriate emphasis on the necessity for accurate time reporting
.nevertheless difficult problem arise in deciding how time should be charged to clients .if the normal work week is 40
hrs should a job be charged for 1/40 th of a week compensation for each other spent on it? If so how should work done
on evening and weekend be counted how to account for time spent reading literature ,going to meeting ,and otherwise
keeping up to date?

4. Small Size
With a few exception such as some law firm and accounting firms ,professional organisations are relatively small and
operate at a single location .senior management in such organisations can personally observe what is going on and
personally motivate employee .thus there is less need for a sophisticated management control system ,with profit
centres and formal performance reports nevertheless even a small organisations need a budget a regular comparison of
performance against budget ,and a way relating compensation to performance.
5. Marketing
In a manufacturing company there is a dividing line between marketing activities and production activities only senior
management is concerned with both .such a clean separation does not exist in most Professional organisation, however
their time and this complicate the track of measuring performance .this reluctant seems to have its root in tradition
usually it can be overcome if senior management is willing to put appropriate emphasis on the necessity for accurate
time reporting. Nevertheless difficult problem arise in deciding how time should be charged to clients .if the normal
work. These marketing activities are conducted by professional usually by professional, usually by professional who
spend much of their time in production work that is working for clients.
In such situation it is difficult to assign appropriate credit to the person responsible for selling a new customer; in a
consulting firm for example a new engagement may result from a conversation between a member of the firm or
from the reputation of one of the firm professional as an outgrowth of speeches or articles. Moreover the profession
al who is responsible for obtaining the engagement may not personally involved in carrying it out .until fairly
recently these marketing contribution were rewarded subjectively that is they were taken into account in promotion
and compensation decisions .some organisation now give explicit credit, perhaps as a percentage of the project
revenue, if the person revenue, if the person who hold sold the project can be identified.

What is Interactive Control?

Interactive control alerts management of strategic uncertainties either trouble or opportunities that become the basis
for manager to adapt to a rapidly changing environments by thinking about new strategies.
1. A subset of the management control information that has a bearing on the strategic uncertainties facing the
buss becomes the focal point.
2. Senior executive take such information seriously.
3. Managers at all levels of the org focus attention on the information produced by the system.

Q5: Shandilya Ltd. (MCS-2008) Numerical


Shandilya Ltd. has adopted Economic Value Added (EVA) technique for the appraisal of performance of its three
divisions A,B and C. Company charges 6% for current assets and 8 % for Fixed Assets, while computing EVA
relevant data are given below :Particulars

Div A
Budgeted

Div B
Actual

Budgeted

Div C
Actual

10

Budgeted

Total
Actual

Budgeted

Actual

Profit

360

320

220

240

200

200

780

760

Current Assets

400

360

800

760

1200

1400

2400

2520

Fixed Assets

1600

1600

1600

1800

2000

2200

5200

5600

Solution:
Particulars

Div A

Div B

Div C

Total

Budgeted

Actual

Budgeted

Actual

Budgeted

Actual

Budgeted

Actual

ROA

18%

16%

9%

9%

6%

6%

10%

9%

EVA

208

170.4

44

50.4

-32

-60

220

160.8

b) Comment upon both methods, based on results.


There are three apparent benefits of an ROA measure. First, it is a comprehensive measure in that anything that
effects the financial statements is reflected in this ratio. Secondly, ROA is easy to calculate, easy to understand, and
meaningful in absolute sense. Finally, it is a common denominator that may be applied to any organizational units
responsible for profitability, no matter what its size or what business it practices. The performance of different units
may be compared directly to each other. Also, ROI data is available for competitors that can be used as a basis for
comparison. Nevertheless, the EVA approach has some inherent advantages over ROA.
There are three compelling reasons to use EVA over ROI. First, with EVA all business units have the same profit
objective for comparable investments. The ROI approach, on the other hand, provides different incentives for
investment across business units. For example, a business unit that is currently achieving 30% ROA would be most
reluctant to expand unless it is able to earn a ROI of 30% or more on additional assets. Second, decision that increase
a centres ROI may decrease its overall profits. Third advantage of EVA is that different interest rates may be used
for different types of assets. For example, a relatively low rate May be used for inventories while a higher rate may
be used for different types of fixed assets.

SET .3
MANAGEMENT CONTROL SYSTEM

Q.1) Describe differences in budgeting perspective of engineered and discretionary expense


centre

1.Expense centers:
Expenses center are responsibility centers for which input or expenses are measured in monetary terms,
but for which outputs are not measured in monetary terms. There are two general types: engineered

11

expense center and discretionary expense center. They correspond to two types of costs.. Engineered costs
are elements of cost for which the right or proper amount of costs that should be incurred can be estimated
with a reasonable degree of reliability. Costs incurred in factory for direct labour direct material
component supplies and utilities are examples.

2.Engineered expense centers:


Engineered expense center have the following characteristics:
1. Their inputs can be measured in monetary ter ms.
2. Their output can be measured in physical terms.
3. The optimal dollar amount of input required to produce one unit of output can be established
Engineered expense center usually are found in manufacturing operations. Warehousing, distribution,
trucking and similar units in the marketing organization also may be engineered expense center and so
many certain responsibility center within administrative and support department. Examples are accounts
receivable account payable and payroll section in the control ler department personnel record and cafeteria
in the human resource department shareholder record in the corporate secretary department and the
company motor pool. Such units perform repetitive task for which standard cost can be developed
In an engineered expense center the output multiplied by the standard cost or each unit produced
represents what the finished product should have cost. When this cost is compared to actual costs, the
difference between the two represents the efficiency of the organizati on unit being measured.
We emphasize that engineered expense centers have other important tasks not measured by cast alone. The
effectiveness of these aspects of performance should be controlled. For example expenses center
supervisor are responsible for the quality of good and for the volume of production in addition to their
responsibility for cost efficiency. Therefore the type and amount of production is prescribed and specific
quality standards are set so that manufacturing costs are not minimized at the expense of quality. Moreover
manager of engineered expense center may be responsible for activities such a training that are not related
to current production judgment about their performance should include an appraisal of how well they carry
out these responsibilities.
There are few if any responsibility center in which all cost items are engineered. Even in highly automated
production department the amount of indirect labour and of various services used can vary with
management discretion.
Thus, the term engineered costs center refers to responsibility center in which engineered cost predominate
but it does not imply that valid engineering estimates can be made for each and every cost item.

3.Discretionary expense center:


The output of discretionary expenses center cannot be measured in monetary terms. They include
administration and support units research and development organization and most marketing activities.
The term discretionary does not mean that management judgments are capricious or ha phazard.
Management has decided on certain policies that should govern the operation of the company. One
company may have a small headquarter staff another company of similar size and in the same industry may
have a staff that is 10 time as large the manag ement of both companies may be concerned that they made
the correct decision on staff size but there is no objective way judging which decision was actually better
manager are hired and paid to make such decision after such a drastic change the level of di scretionary
expenses generally has a similar pattern from one year to the next.
The difference between budgeted and actual expense is not a measure of efficiency in a discretionary
expense centre it is simply the difference between the budgeted input and the actual input. It in no way
measures the value of the output, if actual expense do not exceed the budget amount, the manager has
lived within the budget however ,because by definition the budget does not purport to measure the
optimum amount of spending we cannot say that living within the budget is efficient performance .

12

4.Differences in budgeting perspective of engineered and discretionary expense centre


Budget preparation
The decision that management make about a discretionary expense budget ar e different from the decisions
that it makes about the budget for an engineered expense center. For the latter, management decides
whether the proposed operating budget represent the cost of performing task efficiently for the coming
period. Management is not so much concerned with the magnitude of the task because this is largely
determined by the actions of other responsibility centers, such as the marketing departments ability to
generate sales. In formulating the budget for a discretionary expense cent er, however management
principal task is to decide on the magnitude of the job that should be done.
Incremental budgeting:
Here the current level expenses in a discretionary expense center is taken as a starting points this amount
is adjusted for inflation for anticipated changes in the workload of continuing tasks for special tasks and if
the data are readily available for the cost of comparable work in similar units.
There are two drawbacks to incremental budgeting. First because managers of these cente rs typically want
to provide more service they tend to request additional resources in the budgeting process and if they make
a sufficiently strong case these request will be granted. This tendency is expressed in Parkinsons second
law: overhead costs tend to increase period. There is ample evidence that not all this upward creep in cost
is necessary.
This problem is especially compounded by the fact that the current level of expenditure in the
discretionary expenses center is taken for granted and is no t re-examined during the budget preparation
process. Second when a company faces a crises or when a new management takes over overhead costs are
sometimes drastically reduced without any adverse consequences.
Despite this limitation most budgeting in discretionary expense centers is incremental. Time does not
permit the more thorough analysis described in the next section.

13

Zero based review:


An alternative approach is to make a thorough analysis of each discretionary expense center on a schedule
that will cover all of them over a period of five year or so. That analysis provides a new base. There is a
likelihood that expenses will creep up gradually over the next five years and this is tolerated at the end of
five years, another new base is established. Such an analysis is often called a zero base review.
In contrast with incremental budgeting which takes the current level of spending as the starting point this
more intensive review attempts to build up de now the resources that actually are needed by th e activity.
Basic question are raised;(1) should use customer?(2) what should the quality level be ?are we doing too
much(3)should the function be performed in this way (4) how much should it cost?
Cost variability:
In discretionary expense center costs tend to vary with volume from one year to the next but they tend not
to vary with short run fluctuation in volume within a given year. By contrast costs in engineering expense
center are expected to vary with short run changes in volume. In part this refl ect the fact that volume
changes do have an impact throughout the company even though their actual impact cannot be measures
the ; in part this reflect the fact that volume changes do have an impact throughout the company even
though their actual impact cannot be measured in part this result from a management personnel and
personnel related costs are by far the largest expense item in most discretionary expense center the annual
budget for these center tend to be a constant percentage of budgeted sales volu me.

Q.2) Explain some factors which may influence top management style and the implication of the
top management style on management control.

The management control function in an organization is influenced by the style of senior management. The
style of the chief executive officer affects the management control process in the entire organization.
Similarly, the style of the business unit manager affects the unit's management control process, and the
style of functional department managers affects the management control process in their functional areas.

Differences in Management Styles


Managers manage differently. Some rely heavily on reports and certain formal documents; others prefer
conversations and informal contacts. Some are analytical; others us e trial and error. Some are risk takers;
others are risk averse. Some are process oriented; others are results oriented. Some are long -term oriented;
others are short-term oriented. Some emphasize monetary rewards; others emphasize a broader set of
rewards.
Management style is influenced by the manager's background and personality. Background includes things
like age, formal education, and experience in a given function, such as manufacturing, technology,
marketing, or finance. Personality characteristics include such variables as the manager's willingness to
take risks and his or her tolerance for ambiguity.

Implications for Management Control


The various dimensions of management style significantly influence the operation of the control systems.
Even if the same reports with the same set of data go with the same frequency to the CEO, two CEOs with
different styles would use these reports very differently to manage the business units.
Style affects the management control process how the CEO prefers to use the information, conducts
performance review meetings, and so on which in turn affects how the control system actually operates,
even if the formal structure does not change under a new CEO. In fact, when CEOs change, subordinates
typically infer what the new CEO really wants based on how he or she interacts during the management
control process.

14

Personal versus Impersonal Controls


Presence of personal versus impersonal controls in organizations is an aspect of managerial style.
Managers differ on how much importance they attach to formal budgets and reports as well as informal
conversations and other personal contacts. Some managers are "numbers oriented"; they want a large flow
of quantitative information, and they spend much time analyzing thi s information and deriving tentative
conclusions from it. Other managers are "people oriented"; they look at a few numbers, but they usually
arrive at their conclusions by talking with people, judging the relevance and importance of what they learn
partly on their appraisal of the other person. They visit various locations and spend time talking with both
supervisors and staff to get a sense of how well things are going.
Managers' attitudes toward formal reports affect the amount of detail they want, the f requency of these
reports, and even their preference for graphs rather than tables of numbers, and whether they want
numerical reports supplemented with written comments. Designers of management control systems need to
identify these preferences and accommodate them.

Tight versus Loose Controls


A manager's style affects the degree of tight versus loose control in any situation. The manager of a
routine production responsibility center can be controlled relatively tightly or loosely, and the actual
control reflects the style of the manager's superior. Thus, the degree of tightness or looseness often is not
revealed by the content of the forms or aspects of the formal control documents, rules, or procedures. It is
a factor of how these formal devices are used. The degree of looseness tends to increase at successively
higher levels in the organization hierarchy: higher -level managers typically tend to pay less attention to
details and more to overall results.
The style of the CEO has a profound impact on ma nagement control. If a new senior manager with a
different style takes over, the system tends to change correspondingly. It might happen that the manager's
style is not a good fit with the organization's management control requirements. If the manager reco gnizes
this incongruity and adapts his or her style accordingly, the problem disappears. If, however, the manager
is unwilling or unable to change, the organization will experience performance problems. The solution in
this case might be to change the manager.

Q.3) Explain advantages and disadvantages of two step transfer pricing and profit sharing methods

Transfer pricing: If two or more profit center is jointly responsible for product
development manufacturing and marketing each should share in the re venue that is
generated when the product is finally sold. The transfer price is not primarily an
accounting tool; rather, it is a behavioral tool that motivates manager to make the right
decisions. In particular the transfer price should be designed so tha t it accomplishes the
following objective: It should provide each segment with the relevant information
required to determine the optimum tradeoff between company cost and revenues It should
induce goal congruent decisions that is the system should be so d esigned that decision
improve business unit to earn more profit It should help measure the economic
performance of the individual profit center

Two step pricing: First, a charge is made for each unit sold that is equal to the standard
variable cost of production. Second a periodic charge is made for the buying unit. One or
both of these components should include a profit margin. The two step pricing method
correct this problem by transferring variable cost on a per unit basis, and transferring
fixed cost and profit on a lump sum basis under this method the transfer price for product
A would be 5$ for each unit that unit Y purchases plus $20000 per month for fixed cost.
Plus $10000 per month for profit: if transfer of product A in a certain month are at the
expected amount 5000 units then under the two step method unit y will pay the variable
cost of $25000 plus $30000 for the fixed cost and profit a total of $55000 .this is the same
amount as the amount it would pay unit x if the transfer price is less than 5000 units say
4000unoits.unit y would pay $50000 under the two step methods compared with the

15

$44000 it would pay if the transfer price were $11 per unit. The difference is their transfer
prices were for not using a portion of unit X capacity that it has reserved. Note that under
two step method the company variable cost for product A is identifiable to unit Y variable
cost for the product, and unit Y will make the correct short term marketing decisions. Unit
Y also has information on upstream fixed costs and profit related to product A and it can
use these data for long term decision.The fixed cost calculation in the two step pricing
method is based on the capacity that is reserved for the production of product A that is
sold to unit Y the investment represented by this capacity is allocated to product A. The
return on investment that unit X earns on competitive product is calculated and multiplied
by the investment assigned to the product. In the example we calculated the profit
allowance as a fixed monthly amount. It would be appropriate under some circumstance to
divide the investment into variable and fixed component. Then, a profit allowance based
on a return on investment on variable assets would be added to the standard variable cost
for each unit sold.

16

1.
2.

Profit sharing: If the two step pricing system just described is not feasible, a profit sharing
system might be used to ensure congruence of business unit interest with company interest. This
system operates somewhat as follows.

The product is transferred to the marketing unit at standard variable cost.


After the product is sold, the business units share the contribution earned which is selling price minus
the variable manufacturing and marketing costs.

This method of pricing may be appropriate if the demand for the manufactured product is not steady
enough to warrant the permanent assignment of facilities as in the two step method. In general, this
method accomplished the purpose of making the marketing units interest congruent with the companies.
There are several practical problems in implementing such profit sharing system. First, there can be
arguments over the way contribution is divided between the two profit centers. Which is costly, time
consuming and work against basic reason for decentraliz ation namely autonomy of the business units
mangers. Second, arbitrarily divided up the profit between units does not give valid information on the
profitability of each segment of the organization.
Third since the contribution is not allocated until afte r the sale has been made the manufacturing units
contribution depends upon the marketing units ability to sell and on the actual selling price.
Manufacturing units may perceive this situation to be unfair

Two set of price: in this method, the manufacturing units revenue is credited at the outside sales
price, and the buying unit is charged the total standard costs. The difference is changed to a
headquarter account and eliminated when the business unit statement are consolidated, this
transfer pricing method is sometimes used when there are frequent conflict between the buying
and selling units that cannot be resolved by one of the other method both the buying and selling

There are several disadvantages to the system of having two set of transaction pr ices,
however the sum of the business unit profit is greater than overall company profits, senior
management must be aware of this situation in approved budget for the business units and in
subsequent evaluation of performance against these budget. Also, t his system create an illusion
feeling that business units are making money while in fact the overall company might be losing
after taking account of the debits to headquarter. Further this system might motivate business unit
to concentrate more on internal transfers at the expense of outside sales

The fact that the conflict between the business units would be lessened under this system
could be viewed as a weakness. Sometime, it is better for the headquarter to be aware of the
conflict arising out of transfer prices because such conflict may signal problem in either the
organizational structure or In other management systems. Under the two sets of prices method
these conflicts are smoothed over thereby not alerting senior management to these problems.

Q.4) Discuss special challenges faced in controlling R & D activities and possible management
initiatives

Type of financial control: The financial control exercised in a discretionary expense center is
quite different from that in engineered center the lat ter attempts to minimize operating cost by
setting a standard and reporting actual costs against this standards. The main purpose of a
discretionary expense budget on the other hand is to allow the manager to control Cost for
particular in the planning. Costs are controlled primarily by deciding what task should be
undertaken and what level of effort is appropriate for each. Thus in a discretionary expense center
financial control is primary exercised at the planning stage before the amount are incurred.

Measurement of performance: The primary job of the manager of a discretionary expense center
is to accomplish the desired output spending an amount that is on budget is satisfactory. This is in

17

contrast with the report in an engineered expense center which helps higher management to
evaluate the manger efficiency. If these two types of responsibility center are carefully
distinguished management may treat the performance report for the discretionary expense center
as if it were an indication of efficiency Control over spending can be exercised by requiring that
the manger approved be obtain before the budget is over sometimes a certain percentage of
overrun is permitted without additional approval if the budget really set forth the best estimate of
actual cost there is 50 percent probability that it will overrun and this is the reason that some
latitude is often permitted.

Control problems: The control of R & D centers, which are also discretionary expense center is
difficult for the following at least a semi tangible output reasons.

1.

Results are difficult to measure quantitatively. As contrasted with administrative activities, R&D
usually has at least a semi tangible output in patent, new products, or new processes. Nevertheless, the
relationship of these outputs to inputs is difficult to measure and appraise. A complete product of an
R&D group may require several year of effort; consequently input as stated in an annual budget may
be unrelated to outputs. Even if an output can be identified a reliable est imate of its value often cannot
be made. Even if the value of the output can be calculated, it is usually not possible for management to
evaluate the efficiency of the R&D effort because of its technical nature. A brilliant effort may come
up against an insuperable obstacle, whereas a mediocre effort may, by luck result in a bonanza.

2.

The goal congruence problem in R&D center is similar to that in administrative centers. The research
managers typically want to build the best research organization that mone y can buy, even though this
is more expensive than the company can afford. A further problem is that research people often may
not have sufficient knowledge of the business to determine the optimum direction of the research
efforts.

3.

Research and development can seldom be controlled effectively on an annual basis. A research project
may take year s to reach fruition, and the organization must be built up slowly over a long time period.
The principal cost is for the work force obtaining highly skilled scient ific talented is often difficult,
and short term fluctuation in the work force are in efficient. It is not reasonable, therefore to reduce
R&D costs in years when profits are low and increase them in year when profits are high. R&D should
be looked at as a long term investment not as an activity that varies with short run corporate
profitability.

The R&D continuum:Activities conducted by R&D organization lie along a continuum. At one
extreme is basic research; the other extreme is product testing. Basic re search has two
characteristics: first, it is unplanned management at most can specify the general area that is to
be explored second there is often a very long time lag before basic research result in successful
new product introductions. Financial control system has little value in managing basic research
activities. In some companies, basic research in included as a lump sum in the research program
and budget. In others, no specific allowance is made for basic research as such; there is an
understanding that scientists and engineers can devote part of their time to explorations in
whatever direction they find most interesting, subject only to informal agreement with their
supervisor.For product testing projects, on the other hand, the time and financial re quirement
can be estimated, not as accurately as production activities.

Q.5) Explain problems faced in pricing corporate services provided to business units organized as
Profit Centers
Services are intangible in nature. This characteristic of services mak es it difficult for pricing. Charging
business units for services furnished by corporate staff units becomes challenging work due to intangibility
of services. While pricing corporate services, we exclude the cost of central service staff units over which
business units have no control (e.g., central accounting, public relations, and administration). If these costs

18

are charged at all, they are allocated, and the allocations do not include a profit component. The
allocations are not transfer prices.

We need to consider two types of transfers:


O
O

For central services that the receiving unit must accept but can at least partially control
the amount used.
For central services that the business unit can decide whether or not to use.

Business units may be required to use company staffs for services such as information technology and
research and development. In these situations, the business unit manager cannot control the efficiency
with which these activities are performed but can control the amount of the se rvice received. There are
three schools of thought about such services.
One school holds that a business unit should pay the standard variable cost of the discretionary services. If
it pays less than this, it will be motivated to use more of the service t han is economically justified. On the
other hand, if business unit managers are required to pay more than the variable cost, they might not elect
to use certain services that senior management believes worthwhile from the company's viewpoint. This
possibility is most likely when senior management introduces a new service, such as a new project analysis
program. The low price is analogous to the introductory price that companies sometimes use for new
products.
A second school of thought advocates a price eq ual to the standard variable cost plus a fair share of the
standard fixed costs-that is, the full cost. Proponents argue that if the business units do not believe the
services are worth at least this amount, something is wrong with either the quality or th e efficiency of the
service unit. Full cost represents the company's long run costs, and this is the amount that should be paid.
A third school advocates a price that is equivalent to the market price, or to standard full cost plus a profit
margin. The market price would be used if available (e.g., costs charged by a computer service bureau); if
not, the price would be full cost plus a return on investment. The rationale for this position is that the
capital employed by service units should earn a return just as the capital employed by manufacturing units
does. Also, the business units would incur the investment if they provided their own service.
Optional Use of Services
In some cases, management may decide that business units can choose whether to use central service units.
Business units may procure the service from outside, develop their own capability, or choose not to use the
service at all. This type of arrangement is most often found for such activities as information technology,
internal consulting groups, and maintenance work. These service centers are independent; they must stand
on their own feet. If the internal services are not competitive with outside providers, the scope of their
activity will be contracted or their services may be outsour ced completely.
For example, Commodore Business Machines outsourced one of its central service activities -customer
service-to Federal Express. James Reeder, Commodore's vice president of customer satisfaction, said, "At
that time we didn't have the greatest reputation for customer service and satisfaction. But this was FedEx's
specialty, handling more than 300,000 calls for service each day. Commodore arranged for FedEx to
handle the entire telephone customer service operation from FedEx's hub in Memphis.
After losing $29 million online the previous year, Borders Group turned to rival Amazon.com to manage
its online sales. Borders get to maintain an Internet sales channel and gains the operational effectiveness
provided by Amazon.com while being able to focus on the growth of its bricks and mortar business.
In this situation, business unit managers control both the amount and the efficiency of the central services.
Under these conditions, these central groups are profit centers. Their transfer prices shoul d be based on the
same considerations as those governing other transfer prices.

19

(Numerical) MCS 2004


Division B of Shayana company contracted to buy from Div. A, 20,000 units of a components which
goes into the final product made by Div. B. The transfer price for this internal transaction was set at
Rs. 120 per unit by mutual agreement. This comprises of (per unit) Direct and Variable labour cost
of Rs. 20; Material Cost of Rs.60; Fixed overheads of Rs.20 (lumpsum Rs.4 lacs) and Rs.20 lacs that
Div. A would require for this additional activity. During the year, actual off take of Div. B from Div.
A was 19,600 units. Div. A was able to reduce material consumption by 5% but its budgeted
investment overshot by 10%.
a) As Financial controller of Div. A, compare Actual Vs Budgetred Performance
b) Its implications for Management Control?
Solution:
a)
Particulars

Budgeted
(Rs. Per
Unit)

Budgeted
(Total in Rs.)

For 20,000 Units

Direct and
Variable Labour
Cost
Material Cost
Fixed Overheads
Total Cost
Transfer Price
Profit
Investment
ROI =
Profit/Investment

Actual
(Rs. Per Unit)

Actual
(Total in Rs.)

For 19,600 Units

20

4,00,000

20

3,92,000

60
20
100
120
20
20

12,00,000
4,00,000
20,00,000
24,00,000
4,00,000
20,00,000
20%

57

11,17,200
4,00,000
19,09,200
23,49,200
4,40,000
22,00,000
20%

119.86

Despite of increase in investment by 10%, there is negligible difference in transfer price. Also the sales
have decreased by 400 units. Therefore we can say that additional investment has not achieved any
positive results.

SET-4
Q.1) A)Explain the concept of ROI. What are its advantages?
Return on investment (ROI) is the ratio of profit before tax to the gross investment.
ROI is calculated with the help of the following formula:
ROI = (Pre-Tax Profit/Sales) X (Sales/Net Assets) or (Pre-Tax Profits/Net Assets)
The numerator is profit before tax as reported in the P&L account. The profit should include only the profits arising
out of the normal activities of the division. Unusual items of receipts and expenses should be excluded from the profit
figure. One should also ignore windfalls and income from investments not related to the operations of the division.
Tax is excluded from the numerator because the marginal of the SBU is not responsible for or in control of the tax
paid.
Capital employed can be ascertained from the balance sheet by including fixed and current assets. Assets not
currently put to divisional use should be excluded from the investment base. One also needs to exclude their relative
earnings if any. The company should also exclude intangible assets like goodwill, deferred revenue expenses,
preliminary expenses, etc.
ROI can be improved by:

20

Increasing the profit margin on sales.

Increasing the capital turnover

Increasing both profit margin and capital turnover.

Reducing cost as that adds to the total earnings of the firm.

Increasing the profits by expanding present operations or developing new product line, increasing market
share, etc.

Diversifying, introducing productivity imporevement measures, expansion, replacement of old equipments

Advantages of ROI
o

ROI relates return to the level of investment and not sales as the rate of return is more realistic.

ROI can be decomposed into other variables as shown. These variables have tremendous analytical value.

ROI is an effective tool for inter-firm comparison.

Question 1 (b):
Many experts regard EVA as a concept superior to ROI and yet in certain cases, EVA does not do justice to the
evaluation of investment center. Explain this phenomenon with as illustration.
EVA does not solve all the problems of measuring profitability in an investment center. In particular, it does not
solve the problem of accounting for fixed assets discussed above unless annuity depreciation is also used, and this is
rarely done in practice. If gross book value is used, a business unit can increase its EVA by taking actions contrary to
the interests of the company, as shown in Exhibit 7-3. If net book value is used, EVA will increase simply due to the
passage of time. Furthermore, EVA will be temporarily depressed by new investments because of the high net book
value in the early years. EVA does solve the problem created by differing profit potentials. All business units,
regardless of profitability, will be motivated to increase investments if the rate of return from a potential investment
exceeds the required rate prescribed by the measurement system.
Moreover, some assets may be undervalued when they are capitalized, and others when they are expensed. Although
the purchase cost of fixed assets is ordinarily capitalized, a substantial amount of investment in start-up costs, new
product development, dealer organization, and so forth may be written off as expenses, and, therefore, not appear in
the investment base. This situation applies especially in marketing units. In these units the investment amount may be
limited to inventories, receivables, and office furniture and equipment. When a group of units with varying degrees of
marketing responsibility are ranked, the unit with the relatively larger marketing operations will tend to have the
highest EVA.
In view of all these problems, some companies have decided to exclude fixed assets from the investment base. These
companies make an interest charge for controllable assets only, and they control fixed assets by separate devices.
Controllable assets are, essentially, receivables and inventory. Business unit management can make day-to-day
decisions that affect the level of these assets. If these decisions are wrong, serious consequences can occur-quickly.
For example, if inventories are too high, unnecessary capital is tied up, and the risk of obsolescence is increased;
whereas, if inventories are too low, production interruptions or lost customer business can result from the stockouts.
To focus attention on these important controllable items, some companies, such as Quaker Oats, 17 include a capital
charge for the items as an element of cost in the business unit income statement. This acts both to motivate business
unit management properly and also to measure the real cost of resources committed to these items.
Investments in fixed assets are controlled by the capital budgeting process before the fact and by post completion
audits to determine whether the anticipated cash flows, in fact, materialized. This is far from being completely satisfactory because actual savings or revenues from a fixed asset acquisition may not be identifiable. For example, if a

21

new machine produces a variety of products, the cost accounting system usually will not identify the savings attributable to each product.
The argument for evaluating profits and capital investments separately is that this often is consistent with what senior
management wants the business unit manager to accomplish; namely, to obtain the maximum long-run cash flow
from the capital investments the business unit manager controls and to add capital investments only when they will
provide a net return in excess of the company's cost of funding that investment. Investment decisions, then, are
controlled at the point where these decisions are made. Consequently, the capital investment analysis procedure is of
primary importance in investment control. Once the investment has been made, it is largely a sunk cost and should
not influence future decisions. Nevertheless, management wants to know when capital investment decisions have
been made incorrectly, not only because some action may be appropriate with respect to the person responsible for
the mistakes but also because safeguards to prevent a recurrence may be appropriate.
Q.2 What are the different methods to evaluate the performance of an investment centre? Discuss the merits and
demerits of each? Which method would you recommend?
The following techniques are useful in evaluating the performance of an investment centre:
1. Return on investment (ROI):
The rate of return on investment is determined by dividing net profit or income by the capital employed or
investment made to achieve that profit.
ROI = Profit / Invested capital * 100
ROI consists of two components viz.
Profit margin
Investment turnover
ROI = Net profit / Investment
= (Net profit / Sales) * (Sales / Investment in assets)
It will be seen from the above formula that ROI can be improved by increasing one or both of its components viz. the
profit margin and the investment turnover in any of the following ways:
Increasing the profit margin
Increasing the investment turnover
Increasing both profit margin and investment turnover
Capital employed is taken to be the total of shareholders funds, loans etc
The profit figure used is in calculating ROI is usually taken from the profit and loss account, profit arising out of the
normal activities of the company should only be taken.
Capital employed for the company as a whole can be arrived at as follows:
Share capital of the company

xxx

Reserves and surplus

xxx

Loans (secured/unsecured)

xxx
------

22

xxx
Less: a. Investment outside the business

xxx

b. Preliminary expenses

xxx

c. Debit balance of P & L A/c

xxx

xxx
------xxxx

Merits:
Return on investment analysis provides a strong incentive for optimum utilization of the assets of the company. This
encourages managers to obtain assets that will provide a satisfactory return on investment and to dispose off assets
that are not providing an acceptable return. In selecting amongst alternative long-term investment proposals, ROI
provides a suitable measure for assessment of profitability of each proposal.
Demerits:
ROI analysis is not very suitable for short-term projects and performances. In the initial stages a new investment may
yield a small ROI which may mislead the management. Most likely the rate would improve in course of time when
the initial difficulties are overcome.
The book value of assets decline due to depreciation, the investment base will continuously decrease in value,
causing the rate of return to increase.
2. Residual income:
Residual income can be defined as the operating profit (or income) of the company less the imputed interest on the
assets used by the company. In other words, interest on the capital invested in the company is treated as a cost and
any surplus is the residual income. Residual income is profit minus notional interest charge on capital employed.
Residual income is affected by the size of the organization and therefore will not provide a basis for evaluation of
organizational performance. This is probably the main reason why the management continues to make use of ROI
which is relative measure.
Not all projects start off with positive or sufficiently large positive profits in the early years of a project to produce a
positive increment to residual income.
It has been argued that a more suitable measure of performance for investment centres, which could encourage
managers to be more willing to undertake marginally profitable projects, is residual income.
We recommend RI as a method of evaluating performance of an investment centre. Because when RI is adopted for
evaluation purposes, emphasis is placed on marginal profit amount above the cost of capital rather than on the rate
itself.
Q.3 What are the objectives of Transfer Pricing?
Transfer price if designed appropriately has the following objectives:
It should provide each segment with the relevant information required to determine the optimum trade-off between
company costs and revenues.It should induce goal congruent decisions-i.e. the system should be so designed that
decisions that improve business unit profits will also improve company profits. It should help measure the economic
performance of the individual profit centers. The system should be simple to understand and easy to administer.

23

What is ideal transfer price in the situations of Limited Market Shortage of Capacity in the industry

The ideal transfer price in the situations of Limited Market


By limited market it means that the markets for buying and selling profit centers may be limited.
Even in case of limited market the transfer price that is ideal or satisfies the requirement of a profit center system is
the competitive price. In case if a company is not buying or selling its product in an outside market there are some
ways to find the competitive price. They are as follows:
If published market prices are available, they can be used to establish transfer prices. However, these should be prices
actually paid in the market-place and the conditions that exist in the outside market should be consistent with those
existing within the company.
For example, market prices that are applicable to relatively small purchases are not valid in this case.
Market prices are set by bids. This generally can be done only if the low bidder has a reasonable chance of obtaining
the business. One company accomplishes this by buying about one-half of a particular group of products outside the
company and one-half inside the company. The company then puts all of the products out to bid, but selects one-half
to stay inside. The company obtains valid bids, because low bidders can expect to get some of the business. By
contrast, if a company requests bids solely to obtain a competitive price and does not award the contracts to the low
bidder, it will soon find that either no one bids or that the bids are of questionable value.
If the production profit center sells similar products in outside markets, it is often possible to replicate a competitive
price on the basis of the outside price. If the buying profit center purchases similar products from the outside market,
it may be possible to replicate competitive prices for its proprietary products. This can be done by calculating the cost
of the difference in design and other conditions of sale between the competitive products and the proprietary
products.
Shortage of Capacity in the industry
In this case, the output of the buying profit center is constrained and again company profits may not be optimum.
Some companies allow either buying profit center to appeal a sourcing decision to a central person or committee. In
this scenario a buying profit center could appeal a selling profit centers decision to sell outside.
The person/group would then make a sourcing decision on the basis of the companys best interests. In every case the
transfer price would be the competitive price. In other words, the profit center is appealing only the sourcing
decision.
Even if there are constraints on sourcing, the market price is the best transfer price. If the market price can be
approximated, it is ideal transfer price.

When do you use Cost Based Transfer Pricing?

We use cost-based transfer pricing if there is no way of approximating valid competitive price. Transfer prices may
be set up on the basis of cost plus a profit, even though such transfer prices may be complex to calculate and the
results less satisfactory than a market-based price.
Two aspects need to be considered for cost-based transfer pricing:
The cost basis: The usual basis is the standard cost. Actual costs should not be used because production inefficiencies
will then be passed on to the buying profit center. If the standard costs are used, there is a need to provide an
incentive to set tight standards and to improve standards.
The profit markup: In calculating the profit markup, there also are two decisions:

24

What is the profit markup to be based?

The simplest and most widely used base is percentage of costs. If this base is used, however, no account is taken of
capital required. A conceptually better base is a percentage of investment. But there may be a major practical
problem in calculating the investment applicable to a given product. If the historical cost of the fixed assets is used,
new facilities designed to reduce prices could actually increase costs because old assets are undervalued

What is the level of profit allowed?

The second problem with the profit allowance is the amount of the profit. The conceptual solution is to base the profit
allowance on the investment required to meet the volume needed by the buying profit centers. The investment would
be calculated at a standard level, with fixed assets and inventories at current replacement costs. This solution is
complicated and, therefore, rarely used in practice.
Q.4 (a) Transfer Pricing is not an accounting tool comment with an illustration
If a group has subsidiaries that operate in different countries with different tax rates, manipulating the transfer prices
between the subsidiaries can scale down the overall tax bill of the group. For example the tax rate in Country A is
20% and is 50% in Country B. In the larger interest of the group, it would be advisable to show lower profits in
Country B and higher profits in Country A. For this, the group can adjust the transfer price in such a way that the
profits in Country A increase and that in Country B get reduced. For this the group should fix a very high transfer
price if the Division in Country A provides goods to the Division in Country B. This will maximize the profits in
Country A and minimize the profits in Country B. The reverse will be true if the Division in Country A acquires
goods from the Division in Country B.
There is also a temptation to set up marketing subsidiaries in countries with low tax rates and transfer products to
them at a relatively low transfer price.
Transfer price is viewed as a major international tax issue. While companies indulge in all types of activities to lower
their tax liability, the tax authorities monitor transfer prices closely in an attempt to collect the full amount of tax due.
For this they enter into agreements whereby tax is paid on specific transactions in one country only. But if companies
set unrealistic transfer price to minimize their tax liabilities and the same is spotted by the tax authority, then the
company is forced to pay tax in both countries leading to double taxation.
There have been instances where companies have fixed unrealistic transfer prices. The first case relates to Hoffman
La Roche that imported two drugs Librium and Valium into UK at prices of 437 pounds and 979 pounds per kilo
respectively. While the tax authorities in UK accepted the price, the Monopolies Commission did not accept the
company's argument, since the same drugs were available from an Italian firm for 9 pounds and 28 pounds per kilo.
The company's lawyers argued the case before the Commission on two grounds viz.
1. The price was not set on cost but on what the market would bear and
2. The company had incurred an R&D cost that was included in the price.
These arguments did not go well with the Commission and the company was fined 1.85 million pounds for the
manipulative practices adopted while fixing the transfer price.
The second case is of Nissan. The company had falsely inflated freight charges by 40-60% to reduce the profits. The
manipulation helped the company to hide tax to the tune of 237 million dollars. The next year Nissan was made to
pay 106 million dollars in unpaid tax in the USA because the authorities felt that part of their US marketing profits
were being transferred to Japan, as transfer prices on import of cars and trucks were too high. Interestingly the
Japanese tax authorities took a different view and returned the double tax.
With a view to avoid such cases from recurring, Organisation for Economic Cooperation and Development issued
some guidelines in 1995. These guidelines aim at encouraging world trade. They evolved what came to be known as

25

the arm's length price. The principle states that the transfer price would be arrived at on the basis as if the two .
companies are independent and unrelated. The price is determined through:
Comparable Price Method where the price is fixed on the basis of prices of similar products or an approximation to
one.Gross Margin Method where a gross margin is established and applied to the seller's manufacturing cost.
In spite of all these efforts, it has to be admitted that setting a fair transfer price is not easy. So the onus of proving
the price has been put on the taxpayer who is required to produce supporting documents. If the taxpayer fails to do
this he is required to pay heavy penalty. For example, in USA, failure to provide documentary evidence results in a
40% penalty on the arm's length price. In UK the penalty is to the tune of 100% of any tax adjustment. Other
countries are also in the process of evolving tight norms for the same. Countries across the globe also allow the
taxpayer to enter into an Advance Pricing Agreement whereby dispute can be avoided and so also the costly penalty
of double taxation and penalty.
Q.4.( b) Market Price is ideal transfer price even in limited markets. Comments
By limited market it means that the markets for buying and selling profit centers may be limited.
Even in case of limited market the transfer price that is ideal or satisfies the requirement of a profit center system is
the competitive price. In case if a company is not buying or selling its product in an outside market there are some
ways to find the competitive price. They are as follows:
1. If published market prices are available, they can be used to establish transfer prices. However, these should be
prices actually paid in the market-place and the conditions that exist in the outside market should be consistent with
those existing within the company. For example, market prices that are applicable to relatively small purchases are
not valid in this case.
2.Market prices are set by bids. This generally can be done only if the low bidder has a reasonable chance of
obtaining the business. One company accomplishes this by buying about one-half of a particular group of products
outside the company and one-half inside the company.
The company then puts all of the products out to bid, but selects one-half to stay inside. The company obtains valid
bids, because low bidders can expect to get some of the business. By contrast, if a company requests bids solely to
obtain a competitive price and does not award the contracts to the low bidder, it will soon find that either no one bids
or that the bids are of questionable value.
3.If the production profit center sells similar products in outside markets, it is often possible to replicate a competitive
price on the basis of the outside price.
4.If the buying profit center purchases similar products from the outside market, it may be possible to replicate
competitive prices for its proprietary products. This can be done by calculating the cost of the difference in design
and other conditions of sale between the competitive products and the proprietary products.So we see from the above
arguments that market price is ideal transfer price even in limited markets

SET .5
Q.1) Describe and illustrate significance of human behavior patterns in management control system.
Ans. Management control systems influence human behavior. Good management control systems influence behavior
in a goal congruent manner; that is, they ensure that individual actions taken to achieve personal goals also help to
achieve the organization's goals. The concept of goal congruence, describing how it is affected both by informal
actions and by formal systems.
Senior management wants the organization to attain the organization's goals. But the individual members of the
organization have their own personal goals, and they are not necessarily consistent with those of the organization.

26

The central purpose of a management control system, then, is to ensure a high level of what is called "goal
congruence." In a goal congruent process, the actions people are led to take in accordance with their perceived self
interest are also in the best interest of the organization.
The significance of human behavior patterns in management control system can be explained with the help of
Informal Factors that influence Goal Congruence. In the informal forces both internal and external factors play a
key role.
External Factors

External factors are norms of desirable behavior that exist in the society of which the organization is a part. These
norms include a set of attitudes, often collectively referred to as the work ethic, which is manifested in employees'
loyalty to the organization, their diligence, their spirit, and their pride in doing a good job (rather than just putting in
time). Some of these attitudes are local that is, specific to the city or region in which the organization does its work.
In encouraging companies to locate in their city or state, chambers of commerce and other promotional organizations
often claim that their locality has a loyal, diligent workforce. Other attitudes and norms are industry-specific. Still
others are national; some countries, such as Japan and Singapore, have a reputation for excellent work ethics.

Internal Factors

Culture

The most important internal factor is the organization's own culture-the common beliefs, shared values, norms of
behavior and assumptions that are implicitly and explicitly manifested throughout the organization. Cultural norms
are extremely important since they explain why two organizations with identical formal management control
systems, may vary in terms of actual control. A company's culture usually exists unchanged for many years. Certain
practices become rituals, carried on almost automatically because "this is the way things are done here." Others are
taboo ("we just don't do that here"), although no one may remember why. Organizational culture is also influenced
strongly by the personality and policies of the CEO, and by those of lower-level managers with respect to the areas
they control. If the organization is unionized, the rules and norms accepted by the union also have a major influence
on the organization's culture. Attempts to change practices almost always meet with resistance, and the larger and
more mature the organization, the greater the resistance is.

Management Style

The internal factor that probably has the strongest impact on management control is management style. Usually,
subordinates' attitudes reflect what they perceive their superiors' attitudes to be, and their superiors' attitudes ultimately stem from the CEO.
Managers come in all shapes and sizes. Some are charismatic and outgoing; others are less ebullient. Some spend
much time looking and talking to people (management by walking around); others rely more heavily on written
reports.

27

The Informal Organization


The lines on an organization chart depict the formal relationships-that is, the official authority and responsibilities-of
each manager. The chart may show, for example, that the production manager of Division A reports to the general
manager of Division A. But in the course of fulfilling his or her responsibilities, the production manager of Division
A actually communicates with many other people in the organization, as well as with other managers, support units,
the headquarters staff, and people who are simply friends and acquaintances. In extreme situations, the production
manager, with all these other communication sources available, may not pay adequate attention to messages received
from the general manager; this is especially likely to occur when the production manager is evaluated on production
efficiency rather than on overall performance. The realities of the management control process cannot be understood
without recognizing the importance of the relationships that constitute the informal organization.

Perception and Communication

In working toward the goals of the organization, operating managers must know what these goals are and what
actions they are supposed to take in order to achieve them. They receive this information through various channels,
both formal (e.g., budgets and other official documents) and informal (e.g., conversations). Despite this range of
channels, it is not always clear what senior management wants done. An organization is a complicated entity, and the
actions that should be taken by anyone part to further the common goals cannot be stated with absolute clarity even in
the best of circumstances.
Moreover, the messages received from different sources may conflict with one another, or be subject to differing
interpretations. For example, the budget mechanism may convey the impression that managers are supposed to aim
for the highest profits possible in a given year, whereas senior management does not actually want them to skimp on
maintenance or employee training since such actions, although increasing current profits, might reduce future profitability. The informal factors discussed above have a major influence on the effectiveness of an organizations
management control. The other major influence is the formal systems. These systems can be classified into two
types: (1) the management control system itself and (2) rules, which are described in this section.
The Formal Control System
Rules

We use the word rules as shorthand for all types of formal instructions and controls, including: standing instructions,
job descriptions, standard operating procedures, manuals, and ethical guidelines. Rules range from the most trivial
(e.g., paper clips will be issued only on the basis of a signed requisition) to the most important):e.g., capital
expenditures of over $5 million must be approved by the board' of directors).
Some rules are guides; that is, organization members are permitted, and indeed expected, to depart from them, either
under specified circumstances or when their own best judgment indicates that a departure would be in the best
interests of the organization.
Some rules are positive requirements that certain actions be taken (e.g., fire drills at prescribed intervals). Others are
prohibitions against unethical, illegal, or other undesirable actions. Finally, there are rules that should never be
broken under any circumstances: a rule prohibiting the payment of bribes, for example, or a rule that airline pilots
must never take off without permission from the air traffic controller.
Some specific types of rules are listed below:

Physical Controls

28

Security guards, locked storerooms, vaults, computer passwords, television surveillance, and other physical
controls may be part of the control structure.

Manuals
Much judgment is involved in deciding which rules should be written into a manual, which should be considered
to be guidelines rather than fiats, how much discretion should be allowed, and a host of other considerations.
Manuals in bureaucratic organizations are more detailed than are those in other organizations; large
organizations have more manuals and rules than small ones; centralized organizations have more than
decentralized ones; and organizations with geographically dispersed units performing similar functions (such as
fast-food restaurant chains) have more than do single-site organizations

System Safeguards
Various safeguards are built into the information processing system to ensure that the information flowing
through the system is accurate, and to prevent (or at least minimize) fraud of every sort. These include: crosschecking totals with details, requiring signatures and other evidence that a transaction has been authorized,
separating duties, counting cash and other portable assets frequently, and a number of other procedures described
in texts on auditing.

Task Control Systems


Task control is the process of assuring that specific tasks are carried out efficiently and effectively. Many of
these tasks are controlled by rules. If a task is automated, the automated system itself provides the control.

Q.2) Write short notes on


a.

Concept of profit centre in non-profit organization

b.

Management control in matrix structures

c.

Implications of differentiated strategies on controls.

Ans. a) Concept of profit centre in NPO


By law NPO are allowed to make profit but are restrained from distributing it to owners and management This way
they are non profit making organizations (from the owner's point of view). Such organizations include religious,
charitable and educational trusts. Prime goal of management control systems in such organization is enhancing the
service spread first and if possible then cost control rather and than operating efficiency. On the financial front, they
enjoy many concessions from the government such as taxes, subsidies, grants etc so also they attract special control
from these assisting institutes.
Characteristics:
1. Absent of profit performance measure leads to problems in assessing the efficiency of the organization. If
the organization shows large net income it may be because that NPO may not be providing the services to the extent
possible/ expected. If the organization shows net losses it may show the NPO facing risk of bankruptcy. Hence non
availability of clear-cut performance yardstick makes the problem of control worst.
2. NPO's have contributed capital Plant: NPOs do not have shareholder as its stakeholder. The capital
contribution to the business comes by way of contributions to assets such as building and equipments. Second kind
of contribution could be in the form of monetary assistance, which entitles the organization to reap the interest on it

29

keeping the principal amount intact.


3. Operating Assets represents the resources used for running day to day activities. And the contributed assets
are not allowed to mix up with the operating assets.
4. Fund accounting: NPO need to keep two types of financial statements one set for contributed capital and another
for operating capital. The nature of the contributed capital is beyond control of the management and therefore
management concentrates on controlling the operating assets/investments.
5. Governance: Usually NPO are managed by trusts, who exercise less control on operational matters. Hence
performance control is less demanding from owners' point of view and difficult from the point of view of
management.

These characteristics pose difficulty in pricing of the product/services - what could be appropriate price? Usually it is
set at total/full cost. The more stress expected on allocation of scare resources. Though not stricter control, but a
sense of control can be built among the managers by way of using budgets for various activities and expenses. Non
profit basis makes performance evaluation quite impossible. But one can make the things easier by concentrating on
adherence to costs budgets, and enhancing the service base.

b) Management control in matrix structures


Matrix organizational structure assigns multiple responsibilities to the functional heads. Evaluation of performance of
such organizational entities is very difficult. Though they offer economies of using scares functional staff, it poses
problems of casting the individual responsibility. This form of organization is very complex, from the point of view
of management control system.
At the end we must not forget that the management control system is for the organization and not the organization
exists for management control system. One has to mold and remold the management control system to suit the given
organization structure
A citation by Anthony is worth noting in this regard.
Usually in an advertisement agency, account supervisors are shifted from one
account to another on periodic basis, this practice allows the agency to look at the account from the perspectives of
different executives. However taking in to consideration the time lag of result realization in such services is quite
large. And this may pose problem of performance assessment of a particular executive. This does not mean a control
system designer should insist on abandoning the rotation system of the executives.
Matrix structure offers advantages such as faster decision making process, efficiency and effectiveness but
simultaneously it may pose problems such as added complexity in control function, assignment of responsibility and
authority etc.
c) Implications of differentiated strategies on controls
Different corporate strategies imply the following differences in the context in which control systems need to be
designed: As firms become more diversified, corporate-level managers may not have significant knowledge of, or
experience in, the activities of the company's various business units. If so, corporate-level managers for highly
diversified firms cannot expect to control the different businesses on the basis of intimate knowledge of their
activities, and performance evaluation tends to be carried out at arm's length. Single-industry and related diversified

30

firms possess corporatewide core competencies (on which the strategies of most of the business units are based.
Communication channels and transfer of competencies across business units, therefore, are critical in such firms. In
contrast, there are low levels of interdependence among the business units of unrelated diversified firms. This implies
that as firms become more diversified, it may be desirable to change the balance in control systems from an emphasis
on fostering cooperation to an emphasis on encouraging entrepreneurial spirit.

Strategic planning: given the low level of interdependencies, conglomerates tend to use vertical strategic
planning systems-that is, business units prepare strategic plans & submit to senior management to review &
approve. The horizontal dimension might be incorporated into the strategic planning process in a number of
different ways. First, a group executive might be given the responsibility to develop a strategic plan for the group
as a whole that explicitly identifies synergies across individual business units within the group. Second, strategic
plans of individual business units could have an interdependence section, in which the general manager of the
business unit identifies the focal linkages with other business units and how those linkages will be exploited.
Third, the corporate office could require joint strategic plans for interdependent business units. Finally, strategic
plans of individual business units could be circulated to managers of similar business units to critique and
review. These methods are not mutually exclusive. In fact, several of them could be pursued fruit. fully at the
same time.

Budgeting: The chief executives of single-industry firms may be able to control the operations of subordinates
through informal and personally oriented mechanisms, such as frequent personal interactions. This lessens the
need to rely as heavily on the budgeting system as the tool of control. On the other hand, in a conglomerate it is
nearly impossible for the chief executive to rely on informal interpersonal interactions as a control tool; much of
the communication and control has to be achieved through the formal budgeting stem. This implies the following
budgeting system characteristics in a conglomerate. Business unit managers have somewhat greater influence in
developing their budgets since they, not the corporate office, possess most of the information about their respective product/market environments. Greater emphasis is often placed on meeting the budget since the chief
executive has no other informal controls available.

Transfer Pricing: Transfers of goods and services between business units are more frequent in single-industry
and related diversified firms than in conglomerates. The usual transfer pricing policy in a conglomerate is to give
sourcing flexibility to business units and use arm's-length market prices. However, in a single-industry or a
related diversified firm, synergies may be important, and business units may not be given the freedom to make
sourcing decisions.

Incentive Compensation: The incentive compensation policy tends to differ across corporate strategies in the
following ways-

Use of formulas: Conglomerates, in general, are more likely to use formulas to determine business unit
managers' bonuses; that is, they may base a larger portion of the bonus on quantitative, financial measures, such
as X percent bonus on actual economic value added (EVA) in excess of budgeted EVA. These formula-based
bonus plans are employed because senior management typically is not familiar with what goes on in a variety of
disparate businesses. Senior managers of single-industry and related diversified firms tend to base a larger
fraction of the business unit managers bonus on subjective factors. In many related diversified firms, greater
degrees of interrelationships imply that one unit's performance can be affected by the decisions and actions of
other units. Therefore, for companies with highly interdependent business units, formula-based plans that are tied

31

strictly to financial performance criteria could be counterproductive.


Profitability measures: In the case of unrelated diversified firms, the incentive bonus of the 'business unit
managers tend to be determined primarily by the profitabi1ity of that unit, rather than the profitability of the
firm~ Its purpose is to motivate managers to act as though the business unit were their own company. In contrast,
single-industry and related diversified firms tend to base the incentive bonus of a business unit manager on both
the performance of that unit and the performance of a larger organizational unit (such as the product group to
which the business unit belongs or perhaps even .the overall corporation). When business units are
interdependent, the more the incentive bonus of general managers emphasizes the separate performance of each
unit, the greater the possibility of interunit conflict. On the other hand, basing the bonus of general managers
more on the overall corporate performance is likely to encourage greater interunit cooperation, thereby
increasing managers' motivation to exploit interdependencies rather than their individual results.

Business Unit Strategy: Diversified corporations segment themselves into business units and typically assign
different strategies to the individual business units. Many chief executive officers of multi business organizations
do not adopt a standardized, uniform approach to controlling their business units; instead, they tailor the
approach to each business unit's strategy.The strategy of a business unit depends on two interrelated aspects: (1)
Its mission ("What are its overall objectives?") and (2) its competitive advantage. ("How should the business unit
compete in its industry to accomplish its mission?"). Typically business units choose from four missions: build,
hold, harvest, and divest. The business unit has two generic ways to compete and develop a sustainable
competitive advantage: low cost and differentiation.

Mission The mission for existing business units could be either build, hold, or harvest. These missions constitute
a continuum, with "pure build" at one end and "pure harvest" at the other end. To implement the strategy
effectively, there should be congruence between the mission chosen and the types of controls used. The mission
of the business unit influences the uncertainties that general managers face and the short-term versus long-term
trade-offs they make. Management control systems can be systematically varied to help motivate the man
ager to cope effectively with uncertainty and make appropriate short-term versus long term trade-offs. Thus,
different missions often require systematically different management control systems.

Mission and Uncertainty "Build" units tend to face greater environmental uncertainty than "harvest" units for
several reasons: Build strategies typically are undertaken in the growth stage of the product life cycle, whereas
harvest strategies typically are undertaken in the mature decline stage of the product life cycle. Such factors as
manufacturing process; product technology; market demand; relations with suppliers, buyers, and distribution
channels; number of competitors; and competitive structure change more rapidly and are more unpredictable in
the growth stage than in the mature/decline stage. An objective of a build business unit is to increase market
share. Because the total market share of all firms in an industry is 100 percent, the .battle for market share is a
zero-sum game; thus, a build strategy puts a business unit in greater conflict with its competitors than does a
harvest strategy. Competitors' actions are likely to be unpredictable, and this contributes to the uncertainty that
build business units face. On both the input side and the output side, build managers tend to experience greater
dependencies on external individuals and organizations than do harvest managers. For instance, a build mission
signifies additional capital investment (greater dependence on capital markets), expansion of capacity (greater
dependence on the technological environment), increase in market share (greater dependence on customers and
competitors), increase in production volume (greater dependence on raw material suppliers and labor markets),
and so on. The greater the external dependencies a business unit faces, the greater the uncertainty it

32

confronts.Build business units are often in new and evolving industries; thus, build managers are likely to have
less experience in their industries. This also contributes to the greater uncertainty that managers of build units
face in dealing with external constituencies.

Mission and Time Span The choice of build versus harvest strategies has implications for short-term versus
long-term profit trade-offs. The share-building strategy includes (a) price 'cutting, (b) major R&D expenditures
(to introduce new products), and (c) major market development expenditures. These actions are aimed at
establishing market leadership, but they depress short-term profits. Thus, many decisions that a build unit
manager makes, today may not result in profits until some future period. A harvest strategy, on the other hand,
concentrates on maximizing short-term profits.

Strategic Planning When the environment is uncertain, the strategic planning process is especially important
management needs to think about how to cope with the uncertainties, and this usually requll1 longer-range view
of planning than is possible in the annual budget. If the environment is stable, there may be no strategic planning
process at all or only a broad-brush strategic plan. Thus, the strategic planning process is more critical and more
important for build, as compared with harvest, business units. Nevertheless, some strategic planning of the
harvest business units may be necessary because the company's overall strategic plan must encompass all of its
businesses to effectively balance cash flows. In screening capital investments and allocating resources, the
system may be more quantitative and financial for harvest units. A harvest business unit operates in a mature
industry and does not offer tremendous new investment possibilities. Hence, the required earnings rate for such a
business unit may be relatively high to motivate the manager to search for project with truly exceptional returns.
Because harvest units tend to experience stable environments with predictable products, technologies,
competitors, and customers), discounted cash flow PCF) analysis often can be used more confidently. The
required information used to evaluate investments from harvest units is primarily financial. A build unit,
however, is positioned on the growth stage of the product life cycle. Since the corporate office wants to take
advantage of the opportunities in a growing market, senior management may set a relatively low discount rate,
thereby motivating build managers to forward more investment ideas to corporate office. Given the
product/market uncertainties, financial analysis of some projects from build units may be unreliable. For such
projects, nonfinancial data are more important.

Budgeting The calculational aspects of variance analysis comparing actual results with the budget identify
variances as either favorable or unfavorable. However, a favorable variance does not necessarily imply favorable
performance, nor does an unfavorable variance imply unfavorable performance. The link between a favorable or
unfavorable variance, on the one hand, and favorable or unfavorable performance, on the other hand, depends on
the strategic context of the business unit under evaluation.

Incentive Compensation Syste In designing an incentive compensation package for business unit managers, the
following questions need to be resolved:
1.

1. What should the size of incentive bonus payments be relative to the general manager's base salary?
Should the incentive bonus payments have upper limits?

2.

What measures of performance (e.g., profit, EVA, sales volume, market share, product
development) should be used when deciding the general manager's incentive bonus awards? If multiple
performance measures are employed, how should they be weighted?

3.

How much reliance should be placed on subjective judgments in deciding on the bonus amount?

4.

How frequently (semiannual, annual, biennial, etc.) should incentive awards be made?

33

With respect to the first question, many firms use the principle that the riskier the strategy, the greater the proportion
of the general manager's compensation in bonus compared to salary (the "risk/return" principle). They maintain that
because managers in charge of more uncertain task situations should be willing.to take greater risks, they should have
a higher percentage of their remuneration in the form of an incentive bonus. Thus, "build" managers are more likely
than "harvest" managers to rely on bonuses.
As to the second question, when rewards are tied to certain performance criteria, behaviour ls influenced by the
desire to optimize performance with respect to those criteria. Some performance criteria (cost control, operating
profits, and cash flow from operations) focus more on short-term results, whereas other performance criteria (market
share, new product development, market development, and people development) focus on long-term profitability.
Thus,
linking incentive bonus to short-term criteria tends to promote a short-term focus on the part of the general manager
and, similarly, linking incentive bonus to long-term criteria is likely to promote long-term focus. Considering the
relative differences in time horizons of build and harvest managers, it may not be appropriate to use a single, uniform
financial criterion, such as operating profits, to evaluate the performance of every business unit. A better idea would
be louse multiple performance criteria, with differential weights for each criterion depending on the business unit's
mission.
The third question asks how much subjective judgment should affect bonus amounts. At one extreme, a manager's
bonus might be a strict formula-based plan, with the bonus tied to performance on quantifiable criteria (e.g., X
percent bonus on actual profits in excess of budgeted profits). At the other extreme, a manager's incentive bonus
amounts might be based solely on the superior's subjective judgment or discretion. Alternatively, incentive bonus
amounts might also be based on a combination of formula-based and subjective approaches. Performance on most
long-term criteria (market development, new-product development, and people development) is harder to measure
objectively than is performance along most short-run criteria (operating profits, cash flow from operations, and return
on investment).As already noted, build managers- in contrast with harvest managers, should concentrate more on the
long run, so they typically are evaluated more subjectively than are harvest managers.

As to the final question, the frequency of bonus awards does influence the time horizon of managers. More frequent
bonus awards encourage managers to concentrate on short-term performance since they have the effect of motivating
managers to focus on those facets of the business they can affect in the short run.

Competitive Advantage A business unit can choose to compete. Either as a differentiated player or as a

low-cost player, Choosing a differentiation 'approach, rather than a low-cost approach, increases uncertainty of a
business unit's task environment for three reasons.

1.

Product innovation is more critical for differentiation business units than for low cost business units. This is
partly because a low-cost business unit, with primary emphasis on cost reduction, typically prefers to keep
its product offerings stable over time; a differentiation business unit, with its primary focus on uniqueness &
exclusivity, is likely to engage in greater product innovation.

2.

A low cost business unit typically tend to have narrow product lines to minimize the inventory carry costs as
well as to benefit from scale economies. Differentiation business units on the other hand tend to have a

34

broader set of products to create uniqueness.

3.

Low cost business units typically produce no-frill commodity products& these products succeed primarily
because they have lower prices than competing products. However product differentiation business units
succeed if customers perceive that the products have advantages over competing products. Since the
customer perception is difficult to learn about, & since customer loyalty is subject to change resulting from
actions of competitors or other reasons, the demand for differentiated products is typically more difficult to
predict than the demand for commodities.

Q.3) Which management control practices, if followed, in performance measurement of investment centres are
likely to induce goal congruence, in respect of following assets
d.

(i) Idle

e.

(i) Cash

(ii) Intangible

(iii) Leased

(ii) Receivables

(iii) Inventories

Ans. In some business units, the focus is on profit as measured by the difference between revenues and expenses. In
other business units, profit is compared with the assets employed in earning it. We refer to the latter group of
responsibility centers as investment centers.

Measuring Assets Employed

In deciding what investment base to use to evaluate investment center managers, headquarters asks two questions:
First, what practices will induce business unit managers to use their assets most efficiently and to acquire the proper
amount and kind of new assets? Presumably, when their profits are related to assets employed, business unit
managers will try to improve their performance as measured in this way. \Senior management wants the actions that
they take toward this end to be in the best interest of the whole corporation. Second, what practices best measure the
performance of the unit as an economic entity?

Cash

Most companies control cash centrally because central control permits use of a smaller cash balance than would be
the case if each business unit held the cash balances it needed to weather the unevenness of its cash inflows and
outflows. Business unit cash balances may well be only the "float" between daily receipts and daily disbursements.
Consequently, the actual cash balances at the business unit level tend to be much smaller than would be required if
the business unit were an independent company. Many companies therefore use a formula to calculate the cash to be
included in the investment base. For example, General Motors was reported to use 4.5 percent of annual sales; Du
Pont was reported to use two months' costs of sales minus depreciation.
One reason to include cash at a higher amount than the balance carried by a business unit is that the higher amount is
necessary to allow comparisons to outside companies. If only the actual cash were shown: by internal units would
appear abnormally high and might mislead senior management.
Some companies omit cash from the investment base. These companies reason that the amount of cash approximates

35

the current liabilities; if this is so, the sum of accounts receivable and inventories will approximate the working
capital.

Receivables

Business unit managers can influence the level of receivables, not only indirectly by their ability to generate
sales, and directly, by establishing credit terms by approving individual credit accounts and credit limits, and by
the collecting overdue amount. In the interest of simplicity, receivable included at the actual end-.of-period
balances, although the average of intraperiod balances is conceptually a better measure of the am should be
related to profits.
Whether to include accounts receivable at selling prices or at cost of goods sold is debatable. One could argue
that the business unit's real investment in accounts receivable is only the cost of goods sold and that a satisfactory
return on this investment is probably enough. On the other hand, it is possible to argue that the business unit
could reinvest the money collected from accounts receivable, and, therefore, accounts receivable should be
included at selling prices. The usual practice is to take the simpler alternative-that is, receivables at the book
amount, which is the selling price less an allowance for bad debts.
If the business unit does not control credits and collections, receivables may be calculated on a formula basis. This
formula should be consistent with the normal payment period-for example, 30 days' sales where payment is made
30 days after the shipment of goods.
Inventories
Inventories ordinarily are treated in a manner similar to receivables that is they are often recorded at end-ofperiod amounts even though intraperiod averages would be preferable conceptually. If the company uses LIFO
(last in first out) for financial accounting purposes, a different valuation method usually is used for business unit
profit reporting because LIFO inventory balances tend to be unrealistically low in periods of inflation. In these
circumstances, inventories should be valued at standard or average costs, and these same costs should be used to
measure cost of sales on the business unit income statement
If work-in-process inventory is financed by advance payments or by progress payments from the customer, as is
typically the case with goods that require a long manufacturing period, these payments either are subtracted from the
gross inventory amounts or reported as liabilities.
For e.g. with manufacturing periods a year or greater, Boeing received progress payments for its airplanes
and recorded them as liabilities.
Some companies subtract accounts payable from inventory on the grounds that accounts payable represent financing
of part of the inventory by vendors, at zero cost to the business unit. The corporate capital required for inventories is
only the difference between the gross inventory amount and accounts payable. If the business unit can influence the
payment period allowed by vendors, then including accounts payable in the calculation encourages the manager to
seek the most favorable terms. In times of high interest rates or credit stringency, managers might be encouraged to
consider forgoing the cash discount to have, in effect, additional financing provided by vendors. On the other hand,
delaying payments unduly to reduce net current assets may not be in the company's best interest since this may hurt
its credit rating.

Leased Assets

Suppose the business unit whose financial statements are shown in Exhibit 1 (see page 21) sold its fixed assets for

36

their book value of $300,000, returned the proceeds of the sale to corporate headquarters, and then leased back the
assets at a rental rate of $60,000 per year. As Exhibit 2 (see page 21) shows, the business unit's income before taxes
would decrease because the new rental expense would be higher than the depreciation charge that was eliminated.
Nevertheless, economic valued added would increase because the higher cost would be more than offset by the
decrease in the capital charge. Because of this, business unit managers are induced to lease, rather than own, assets
whenever the interest charge that is built into the rental cost is less than the capital charge that is applied to the
business unit's investment base. (Here, as elsewhere, this generalization oversimplifies because, in the real world, the
impact of income taxes must also be taken into account.)
Many leases are financing arrangements-that is, they provide an alternative way of getting to use assets that otherwise
would be acquired by funds obtained from debt and equity financing. Financial leases (i.e., long-term leases
equivalent to the present value of the stream of lease charges) are similar to debt and are so reported on the balance
sheet. Financing decisions usually are made by corporate headquarters. For these reasons, restrictions usually are
placed on the business unit manager's freedom to lease assets.

Idle Assets

If a business unit has idle asset that can be used by other units, the business unit may be permitted to exclude them
from the investment base if it classifies them as available. The purpose of this permission is to encourage business
unit managers to release underutilized assets to units that may have better use for them. However, if the fixed assets
cannot be used by other units, permitting the business unit manager to remove them from the investment base could
result in dysfunctional actions For example; it could encourage the business unit manager to idle partially utilized
assets that are not earning a return equal to the business unit's profit objective. If there is no alternative use for the
equipment, any contribution from this equipment will improve company profits.

Intangible Assets

Some companies tend to be R&D intensive (e.g., pharmaceutical firms such as Novartis spend huge amounts on
developing new products); others tend to be marketing intensive (e.g., consumer products firms such as Unilever
spend huge amounts on advertising). There are advantages to capitalizing intangible assets such as R&D and
marketing and then amortizing them over a selected life. This method should change how the business unit manager
views these expenditures. By accounting for these assets as long-term investments, the business unit manager will
gain less short-term benefit from reducing out lays on such item, For instance, if R&D expenditures are expensed
immediately, each dollar of R&D cut would be a dollar more in pretax profits. On the other hand, if R&D costs are
capitalized, each dollar cut will reduce the assets employed by a dollar; the capital charge is thus reduced only by one
dollar times the cost of capital, which has a much smaller positive impact on economic valued added.

SET.6
Q.1) What do you understand by Goal Congruence? What are the informal factors that influence goal
congruence?

Ans: This term is used when the same goals are shared by top managers and their subordinates. This is one of the
many criteria used to judge the performance of an accounting system. The system can achieve its goal more
effectively and perform better when organizational goals can be well aligned with the personal and group goals of
subordinates and superiors. The goals of the company should be the same as the goals of the individual business
segments. Corporate goals can be communicated by budgets, organization charts, and job descriptions.

37

Goal Congruence- Meaning Individuals work in different hierarchies and handle different responsibilities

& may have different goals. But they must come together as far as Companys Goal is concerned (there action must
speak Cos language.)

Goal Congruence
Example 1 The HR manager has devised a HR training program to enhance the skills of its sales personnel, with an
objective to enhance their productivity But if company is in strategic need of attaining a certain sales volume in a
given quarter, it can not do so on account of non availability of personnel.
Example 2 The marketing department has planned an impressive advertising campaign, which promises good
returns, But say due to cash crunch Companys current financial position may not let to lose the strings
Example 3 Production Manager may get a good applause for reducing cycle time; But at what cost? Building up the
high inventory i.e. higher investment in current assets. While doing so he just overlooked the financial interest of the
company. After completing the given activity in more efficient manner the concerned manager scores the point/s on
his score card. Whether his actions are leading to scoring of points on the organizations score card too? if it is so
then only one can say the organization is marching towards a common goal.
Every individual working in an organization has got his own motive to do the work. Individuals act in their own
interest, based on their own motivations. And it is always not necessarily consistent with the Cos goal. In a goal
congruence process, the actions the people are led to take in accordance with their perceived self interest are also in
the best interest of the organization i.e. Goal congruence ensures that the action of manager taken in their best interest
is also in the best interest of the organization.

Informal factors that influence goal congruence:


Informal Factors
External factors set of attitudes of the society, work ethics of the society
Internal factors (Factors within the organization)

Culture-Common beliefs, shared values, norms of behavior & assumptions

Implicitly accepted and explicitly built into.

Mgt. Style Informal/Formal

The Communication Channels

Perception and Communication e.g. Budget (meaning) strict profit.

Organizations with Business Divisions (Profit Centre) format have observed that Divisional

Controllers experience divided loyalty in carrying out their functions, causing a possible dysfunction. How
could such a situation be resolved? Define role of controller which suits your suggestion.

To the extent the decision are decentralized top management may lose some control. Relying on control reports is not
as effective as personal knowledge of an operation. With profit center, top management must change its approach to
control. Instead of personal direction senior management must rely to a considerable extent on management control
reports.

38

Competent units that were once cooperating as functional units may now compete with one another dis
advantageously. An increase in one managers profit may decrease those of another. This decrease in cooperation
may manifest itself in a manager unwillingness to refer sales lead to another business unit, even though that unit is
better qualified to follow up on the lead in production decision that have undesirable cost consequence on other units
or in the hoarding of personnel or equipment that from the overall company standpoint would be better off used in
another units.
There may be too much emphasis on short run profitability at the expense of long run profitability. In the desire to
report high current profits, the profit center manager may skip on R&D, training, maintenance. This tendency is
especially prevalent when the turnover of profit center managers is relatively high. In these circumstances, manager
may have good reason to believe that their action may not affect profitability until after they have moved to other job.
There is no complete satisfactory system for ensuring that each profit center by optimizing its own profit , will
optimize company profits.
If headquarter management is more capable or has better information then the average profit center manager the
quality of some of the decision may be reduced.
Divisionalization may cause additional cost because it may require additional management staff personnel and
recordkeeping and may lead to redundant at each profit center.
Business units as profit centers:
Business units are usually set up at profit centers. Business unit managers tend to control product development,
manufacturing, and marketing resources. They are in a position to influence revenue and cost and as such can be held
accountable for the bottom line. However as pointed out in the next section a business unit manager authority may be
constrained such constrained should be incorporated in designing and operating profit center.
Constraint on business unit authority
To realize fully the advantage of the profit center concept the business unit manger would have to be as autonomous
as the president of the independent company. As a practical matter however such autonomy is not feasible. If a
company were divided into completely independent units the organization would be giving up the advantage of size
and synergism. Also senior management authority that a board of director gives to the chief executive. Consequently
business unit structure represents trade off between business unit autonomy and corporate constraint. The
effectiveness of a business units organization is largely dependent on how well these trade off are made.
The performance of a profit center is appraised by comparing actual results for one or more orf these measures with
budgeting amounts. In addition, data on competitors and the industry provide a good cross check on the appropriate
of the budget. Data for individual companies are available from the securities and exchange commission for about
key business ratios; standard & poor computer services, Inc; Robert Morris associates annual statement studies; and
annual survey published in fortune, business week, and Forbes. Trade associations publish data for the companies in
their industries.
Revenues: choosing the appropriate revenue recognition method is important. Should revenue be recognized at the
time as order is received, at the time an order is shipped, or at the time cash is received?
In addition to that decision, issues related to common revenues may need to be considered. There are some situations
in which two or more profit centers participate in the sales effort that results in a sale; ideally, each should be given
appropriate credit for its part in this transaction. Many companies have not given much attention to the solution of
these common revenue problems. They take the position that the identification of price responsibility for revenue
generation is too complicated to be practical and that sale personnel must recognize they are working not only for

39

their own profit center but also for the overall good of the company. They for example, may credit the business unit
that takes an order for a product handled by the another unit with the equivalent of a brokerage commission or a
finder fee. In the case of a bank the branch performing a service may be given explicit credit for that service even
though the customer account is maintained in another branch.
Role of controller

It should publish procedure and forms for the preparation of the budget.

It should provide assistance to budgetees in the preparation of their budget.

It should administer the process of making budget revision during the year.

It should coordinate the work of budget departments in lower echelons

It should analyze reported performance against budget, interprets the result, and prepares summary report for
senior management.

Part of a multinational group, Sundaram Shoe Company(SSC), established its own facilities in India over
75 years ago and enjoyed an excellent record-high market share for its diverse range of shoes, growth and
profits. SC markets its products through company owned shops and its own personnel. Organization
structure is functional. Since 2001, profitability, market share are slipping. Pressure from cheap Chinese
shoes and also premium shoes like Nike has made the company think< of organizational restructuring and
introducing Comensurate Control System to regain its position. Although SSC outsources, 30% of
products, it is seen as a production oriented company. SSC wants to adopt measures to reduce costs,
strengthen marketing and be in a position to produce and meet unexpected and unusual customer
demands. How should the company reorganize to achieve Goal Congruence. Define Performance Metric?

In a goal congruent process, the actions people are led to take in accordance with their perceived self-interest are also
in the best interest of the organization. A firms strategy has a major influence on its structure. The type of structure
in turn influences the design of the organizations management control systems. Sundaram Shoe Companys (SSC)
organization structure is functional which involves the notion of a manager who brings specialized knowledge to bear
on decisions related to a specific function, vis--vis a general purpose manager who lacks the specialized knowledge.
A skilled marketing and production manager would be able to make better decisions in their respective fields. He
would also be able to supervise workers in the same function better than the generalist would. Thus an important
advantage of the functional structure is efficiency. A major disadvantage of this structure is that there is no
unambiguous way of determining the effectiveness of the separate functional managers because each function
contributes jointly to the organizations final output. Therefore, there is no way of determining how much of the
profit was earned respectively by the several production departments.
Sundaram Shoe Company which was a market leader for a period of over 75 years has been losing market share,
which has impacted its profitability. Also it needs to be seen that the company outsources about 30% of its products.
The company aims to strengthen marketing, reduce costs and wants to be in a position to customize products as per
the demands of the customer. Thus, Sundaram needs to re-organize its organization structure which is functional to a
Business Unit form of organization. The benefits of the re-organization would be that the business unit or the division
would be responsible for all the functions involved in producing and marketing a specified product line. The business
managers act almost as if their units are separate companies. They are responsible for planning and co-coordinating

40

the work of the separate functions. Their performance is measured by the profitability of the business unit. This is a
valid criterion because profit reflects the activities of both marketing and production.
Though business unit managers exercise broad authority over their units, headquarters reserves certain key
prerogatives. Headquarters are responsible for obtaining funds for the company as a whole and allocating it to the
business unit, as well as approving budgets and judging the performance of business unit managers, setting their
compensation.
A major advantage of the Business unit structure of organization is that because it is close to the market for its
products than the headquarters, its manager may make sounder production and marketing decisions than headquarters
might and the unit as a whole reacts to new threats or opportunities quickly. This re-organization would help in
achieving goal congruence in the organization.
Performance Metrics are high-level measures what you are doing; that is, they assess your overall performance in the
areas you are measuring. They are external in nature and are most closely tied to outputs, customer requirements, and
business needs for the process.
The performance measurement system should cover the following areas at a minimum:
CUSTOMERS
1.

Performance against customer requirements

2.

Customer Satisfaction

PERFORMANCE OF INTERNAL WORK PROCESSES


1.

Cycle times

2.

Product and service quality

3.

Cost performance (could be productivity measures, inventory, etc.)

SUPPLIERS
1.

Performance of suppliers against your requirements

FINANCIAL
1.

Profitability (could be at the company, product line, or individual level)

2.

Market share growth and other standard financial measures

EMPLOYEE
1.

Associate satisfaction

SET .7
Q: 1) (A) Describe the factors which impact service organization
Ans: Factors which impact service organization:

Absence of Inventory Buffer:

Goods can be held as inventory, which is a buffer that dampens the impact on production activity of fluctuations in
sales volume. Services cannot be stored. The airplane seat, hotel room, hospital operating room, or the hours of

41

lawyers, physicians, scientists, and other professionals that are not used today are gone forever. Thus, although a
manufacturing company can earn revenue in the future from products that are on hand today, a service company
cannot do so. It must try to minimize its unused capacity.

Moreover, the costs of many service organizations are essentially fixed in the short run. In the short run, a hotel
cannot reduce its costs substantially by closing off some of its rooms. Accounting firms, law firms, and other
professional organizations are reluctant to layoff professional personnel in times of low sales volume because of the
effect on morale and the costs of rehiring and training.

Difficulty in Controlling Quality:

A manufacturing company can inspect its products before they are shipped to the consumer, and their quality can be
measured visually or with instruments (tolerances, purity, weight, color, and so on). A service company cannot judge
product quality until the moment the service is rendered, and then the judgments are often subjective. Restaurant
management can examine the food in the kitchen, but customer satisfaction depends to a considerable extent on the
way it is served. The quality of education is so difficult to measure that few educational organizations have a formal
quality control system.

Labor Intensive:

Manufacturing companies add equipment and automate production lines, thereby replacing labor and reducing costs.
Most service companies are labor intensive and cannot do this. Hospitals do add expensive equipment, but mostly to
provide better treatment, and this increases costs. A law firm expands by adding partners and new support personnel.

Multi-Unit Organizations:

Some service organizations operate many units in various locations; each unit relatively small. These organizations
are fast-food restaurant chains, auto rental companies, gasoline service stations, and many others. Some of the units
are owned; others operate under a franchise. The similarity of the separate units provides a common basis for
analyzing budgets and evaluating performance not available to the manufacturing company. The information for each
unit can be compared with system wide or regional averages, and high performers and low performers can be
identified. However because units differ in the mix of services they provide, in the resources that they use, and in
other ways, care must be taken in making such comparisons.

Q:1) (B) Explain special characteristics of professional organization which would have a bearing on their
control system.

Ans: Special Characteristics of Professional Organization:

Goals:

A dominant goal of a manufacturing company is to earn a satisfactory profit, specifically a satisfactory return on
assets employed. A professional organization has relatively few tangible assets; its principal asset is the skill of its
professional staff, which doesn't appear on its balance sheet. Return on assets employed, therefore, is essentially
meaningless in such organizations. Their financial goal is to provide adequate compensation to the professionals.

42

In many organizations, a related goal is to increase their size. In part, this reflects the natural tendency to associate
success with large size. In part, it reflects economies of scale in using the efforts of a central personnel staff and units
responsible for keeping the organization up to- date. Large public accounting firms need to have enough local offices
to enable them to audit clients who have facilities located throughout the world.

Professionals:

Professional organizations are labor intensive, and the labor is of a special type. Many professionals prefer to work
independently, rather than as part of a team. Professionals who are also managers tend to work only part time on
management activities; senior partners in an accounting firm participate actively in audit engagements; senior
partners in law firms have clients. Education for most professions does not include education in management, but
quite naturally stresses the skills of the profession, rather than management; for this and other reasons, professionals
tend to look down on managers. Professionals tend to give inadequate weight to the financial implications of their
decisions; they want to do the best job they can, re- I regardless of its cost. This attitude affects the attitude of support
staffs and nonprofessionals in the organization; it leads to inadequate cost control.

Output and Input Measurement:

The output of a professional organization cannot be measured in physical terms, such as units, tons, or gallons. We
can measure the number of hours a lawyer spends on a case, but this is a measure of input, not output. Output is the
effectiveness of the lawyer's work, and this is not measured by the number of pages in a brief or the number of hours
in the courtroom. We can measure the number of patients a physician treats in a day, and even classify these visits by
type of complaint; but this is by no means equivalent to measuring the amount or quality of service the physician has
provided. At most, what is measured is the physician's efficiency in treating patients, which is of some use in
identifying slackers and hard workers. Revenues earned is one measure of output in some professional organizations,
but these monetary amounts, at most, relate to the quantity of services rendered, not to their quality (although poor
quality is reflected in reduced revenues in the long run).
Furthermore, the work done by many professionals is non repetitive. No two consulting jobs or research and
development projects are quite the same. This makes it difficult to plan the time required for a task, to set reasonable
standards for task performance, and to judge how satisfactory the performance was. Some tasks are essentially
repetitive: the drafting of simple wills, deeds, sales contracts, and similar documents; the taking of a physical
inventory by an auditor; and certain medical and surgical procedures. The development of standards for such tasks
may be worthwhile, although in using these standards, unusual circumstances that affect a specific job must be taken
into account.

Small Size:

With a few exceptions, such as some law firms and accounting firms, professional organizations are relatively small
and operate at a single location. Senior management in such organizations can personally observe what is going on
and personally motivate employees. Thus, there is less need for a sophisticated management control system, with
profit centers and formal performance reports. Nevertheless, even a small organization needs a budget, a regular
comparison of performance against budget, and a way of relating compensation to performance.

Marketing:

In a manufacturing company there is a clear dividing line between marketing activities and production activities; only
senior management is concerned with both. Such a clean separation does not exist in most professional organizations.
In some, such as law, medicine, and accounting, the profession's ethical code limits the amount and character of overt
marketing efforts by professionals (although these restrictions have been relaxed in recent years). Marketing is an

43

essential activity in almost all organizations, however. If it can't be conducted openly, it takes the form of personal
contacts, speeches, articles, conversations on the golf course, and so on. These marketing activities are conducted by
professionals, usually by professionals who spend much of their time in production work-that is, working for clients.
In this situation, it is difficult to assign appropriate credit to the person responsible for "selling" a new customer. In a
consulting firm, for example, a new engagement may result from a conversation between a member of the firm and
an acquaintance in a company, or from the reputation of one of the firm's professionals as an outgrowth of speeches
or articles. Moreover, the professional who is responsible for obtaining the engagement may not be personally
involved in carrying it out. Until fairly recently, these marketing contributions were rewarded subjectively- that is,
they were taken into account in promotion and compensation decisions. Some organizations now give explicit credit,
perhaps as a percentage of the project's revenue, if the person who "sold" the project can be identified.

Q:2) Every SBU is a profit center but every profit center is not a SBU? What are the conditions that should
be fulfill for an organization unit to be converted into a profit center? What are the different ways to measure
the performance of profit center? Discuss their relevant merits and demerits.
Ans: Conditions for an organization to be converted into a profit centre: Many management decisions involve
proposals to increase expenses with the expectation of an even greater increase in sales revenue. Such decisions are
said to involve expense/revenue trade-offs. Additional advertising expense is an example. Before it is safe to delegate
such a trade-off decision to a lower-level manager, two conditions should exist.

The manager should have access to the relevant information needed for making such a decision.

There should be some way to measure the effectiveness of the trade-offs the manager has made.

A major step in creating profit centers is to determine the lowest point in an organization where these two conditions
prevail. All responsibility centers fit into a continuum ranging from those that clearly should be profit centers to those
that clearly should not. Management must decide whether the advantages of giving profit responsibility offset the
disadvantages, which are discussed below. As with all management control system design choices, there is no clear
line of demarcation.

Ways to Measure Performance:

There are two types of profitability measurements used in evaluating a profit center, just as there are in evaluating an
organization as a whole. First, there is the measure of management performance, which focuses on how well the
manager is doing. This measure is used for planning, coordinating, and controlling the profit center's day-to-day
activities and as a device for providing the proper motivation for its manager. Second, there is the measure of
economic performance, which focuses on how well the profit center is doing as an economic entity. The messages
conveyed by these two measures may be quite different from each other. For example, the management performance
report for a branch store may show that the store's manager is doing an excellent job under the circumstances, while
the economic performance report may indicate that because of economic and competitive conditions in its area the
store is a losing proposition and should be closed. .

The necessary information for both purposes usually cannot be obtained from a single set of data. Because the
management report is used frequently, while the economic report is prepared only on those occasions when economic
decisions must be made, considerations relating to management performance measurement have first priority in
systems design-that is, the system should be designed to measure management performance routinely, with economic
information being derived from these performance reports as well as from other sources.

44

Types of Profitability Measures

A profit center's economic performance is always measured by net income (i.e., the income remaining after all costs,
including a fair share of the corporate overhead, have been allocated to the profit center). The performance of the
profit center manager, however, may be evaluated by five different measures of profitability: (1) contribution margin,
(2) direct profit, (3) controllable profit, (4) income before income taxes, or (5) net income
(1) Contribution Margin:
Contribution margin reflects the spread between revenue and variable expenses. The principal argument in favor of
using it to measure the performance of profit center managers is that since fixed expenses are beyond their control,
managers should focus their attention on maximizing contribution. The problem with this argument is that its
premises are inaccurate; in fact, almost all fixed expenses are at least partially controllable by the manager, and some
are entirely controllable. Many expense items are discretionary; that is, they can be changed at the discretion of the
profit center manager. Presumably, senior management wants the profit center to keep these discretionary expenses
in line with amounts agreed on in the budget formulation process. A focus on the contribution margin tends to direct
attention away from this responsibility. Further, even if an expense, such as administrative salaries, cannot be
changed in the short run, the profit center manager is still responsible for controlling employees' efficiency and
productivity.
(2) Direct Profit:
This measure reflects a profit center's contribution to the general overhead and profit of the corporation. It
incorporates all expenses either incurred by or directly traceable to the profit center, regardless of whether or not
these items are within the profit center manager's control. Expenses incurred at headquarters, however, are not
included in this calculation. A weakness of the direct profit measure is that it does not recognize the motivational
benefit of charging headquarters costs.
(3) Controllable Profit:
Headquarters expenses can be divided into two categories: controllable and non controllable. The former category
includes expenses that are controllable, at least to a degree, by the business unit manager-information technology
services, for example. If these costs are included in the measurement system, profit will be what remains after the
deduction of all expenses that may be influenced by the profit center manager. A major disadvantage of this measure
is that because it excludes non controllable headquarters expenses it cannot be directly compared with either
published data or trade association data reporting the profits of other companies in the industry.
(4) Income before Taxes:
In this measure, all corporate overhead is allocated to profit centers based on the relative amount of expense each
profit center incurs. There are two arguments against such allocations. First, since the costs incurred by corporate
staff departments such as finance, accounting, and human resource management are not controllable by profit center
managers, these managers should not be held accountable for them. Second, it may be difficult to allocate corporate
staff services in a manner that would properly reflect the amount of costs incurred by each profit center.

There are, however, three arguments in favor of incorporating a portion of corporate overhead into the profit centers'
performance reports. First, corporate service units have a tendency to increase their power base and to enhance their
own excellence without regard to their effect on the company as a whole. Allocating corporate overhead costs to
profit centers increases the likelihood that profit center manager will question these costs, thus serving to keep head
office spending in check. (Some companies have actually been known to sell their corporate jets because of

45

complaints from profit center managers about the cost of these expensive items.) Second, the performance of each
profit center will become more realistic and more readily comparable to the performance of competitors who pay for
similar services. Finally, when managers know that their respective centers will not show a profit unless all-costs,
including the allocated share of corporate overhead, are recovered, they are motivated to make optimum long-term
marketing decisions as to pricing, product mix, and so forth, that will ultimately benefit (and even ensure the viability
of) the company as a whole.
If profit centers are to be charged for a portion of corporate overhead, this item should be calculated on the basis of
budgeted, rather than actual, costs, in which case the "budget" and "actual" columns in the profit center's performance
report will show identical amounts for this particular item. This ensures that profit center managers will not complain
about either the arbitrariness of the allocation or their lack of control over these costs, since their performance reports
will show no variance in the overhead allocation. Instead, such variances would appear in the reports of the
responsibility center that actually incurred these costs. .
(5) Net Income:
Here, companies measure the performance of domestic profit centers according to the bottom line, the amount of net
income after income tax. There are two principal arguments against using this measure: (1) after tax income is often a
constant percentage of the pretax income, in which case there would be no advantage in incorporating income taxes,
and (2) since many of the decisions that affect income taxes are made at headquarters, it is not appropriate to judge
profit center managers on the consequences of these decisions. There are situations, however, in which the effective
income tax rate does vary among profit centers. For example, foreign subsidiaries or business units with foreign
operations may have different effective income tax rates. In other cases, profit centers may influence income taxes
through their installment credit policies, their decisions on acquiring or disposing of equipment, and their use of other
generally accepted accounting procedures to distinguish gross income from taxable income. In these situations, it
may be desirable to allocate income tax expenses
to profit centers not only to measure their economic profitability but also to motivate managers to minimize tax
liability.
Merits:

The quality of decisions may improve because they are being made by managers closest to the point of
decision.

The speed of operating decisions may be increased since they do not have to be referred to corporate
headquarters. . Headquarters management, relieved of day-to-day decision making, can concentrate on
broader issues.

Managers, subject to fewer corporate restraints, are freer to use their imagination and initiative.Because
profit centers are similar to independent companies, they provide an excellent training ground for general
management. Their managers gain experience in managing all functional areas, and upper management
gains the opportunity to evaluate their potential for higher-level jobs.

Profit consciousness is enhanced since managers who are responsible' for profits will constantly seek ways
to increase them. (A manager responsible for marketing activities, for example, will tend to authorize
promotion expenditures that increase sales, whereas a manager responsible for profits will be motivated to
make promotion expenditures that increase profits.).

46

Profit centers provide top management with ready-made information on the profitability of the company's
individual components. . Because their output is so readily measured, profit centers are particularly
responsive to pressures to improve their competitive performance.

Demerits:

Decentralized decision making will force top management to rely more on management control
reports than on personal knowledge of an operation, entailing some loss of control.

If headquarters management is more capable or better informed than the average profit center
manager, the quality of decisions made at the unit level may be reduced.

Friction may increase because of arguments over the appropriate transfer price, the assignment of
common costs, and the credit for revenues that were formerly generated jointly by two or more
business units working together.

Organization units that once cooperated as functional units may now be in competition with one
another. An increase in profits for one manager may mean a decrease for another. In such situations, a
manager may fail to refer sales leads to another business unit better qualified to pursue them; may
hoard personnel or equipment that, from the overall company standpoint, would be better off used in
another unit; or may make production decisions that have undesirable cost consequences for other
units.

Divisionalization may impose additional costs because of the additional management, staff personnel,
and record keeping required, and may lead to task redundancies at each profit center.

Q:3) what are different types of Strategic Missions at SBU level? How do these missions affect Strategic
Planning process and Budgeting at SBU Level?
Different Types of Strategic Missions:
Business Unit Mission:
In a diversified firm one of the important tasks of senior management is resource deployment, that is, make decisions
regarding the use of the cash generated from some business units to finance growth in other business units. Several
planning models have been developed to help corporate level managers of diversified firms to effectively allocate
resources. These models suggest that a firm has business units in several categories, identified by their mission; the
appropriate strategies for each category differ. Together, the several units make up a portfolio, the components of
which differ as to their risk/reward characteristics just as the components of an investment portfolio differ. Both the
corporate 'office and the business unit general manager are involved in identifying the missions of individual
business units. Of the many planning models, two of the most widely used are Boston Consulting Group's two-bytwo growth-share matrix and General Electric Company/McKinsey & Company's three-by-three industry
attractiveness-business strength matrix. While these models differ in the methodologies they use to develop the most
appropriate missions for the various business units, they have the same set of missions from which to choose: build,
hold, harvest, and divest.

Build: This mission implies an objective of increased market share, even at the expense of short-term
earnings and cash flow (e.g., Merck's bio-technology, Black and Decker's handheld electric tools).

Hold: This strategic mission is geared to the protection of the business unit's market share and competitive

position (e.g.: IBM's mainframe computers).

47

Harvest: This mission has the objective of maximizing short-term earnings and cash flow, even at the

expense of market share (e.g., American Brands' tobacco products, General Electric's and Sylvania's light bulbs)

Divest: This mission indicates a decision to withdraw from the business either through a process of slow

liquidation or outright sale. While the planning models can aid in the formulation of missions, they are not cook
books. A business unit's position on a planning grid should not be the sole basis for deciding its mission.

Business Unit Competitive Advantage: Every business unit should develop a competitive advantage in

order to accomplish its mission. Three interrelated questions have to be considered in developing the business unit's
competitive= advantage. First, what is the structure of the industry in which the business unit operates? Second, how
should the business unit exploit the industry's structure? Third, what will be the basis of the business unit's
competitive advantage?

Industry Analysis: Research has highlighted the important role industry conditions play in the performance

of individual firms. Studies have shown that average industry profitability is, by far, the most significant predictor of
firm performance. According to Porter, the structure of an industry should be analyzed in terms of the collective
strength of five competitive forces.
1. The intensity of rivalry among existing competitors. Factors affecting direct rivalry are industry growth, product
differentiability, number and diversity of competitors, level of fixed costs, intermittent overcapacity, and exit
barriers.
2. The bargaining power of customers. Factors affecting buyer power are number of buyers, buyer's switching costs,
buyer's ability to integrate backward, impact of the business unit's product on buyer's total costs, impact of the
business unit's product on buyer's product quality/ performance, and significance of the business unit's volume to
buyers.
3. The bargaining power of suppliers. Factors affecting supplier power are number of suppliers, supplier's ability to
integrate forward, presence of substitute inputs, and importance of the business unit's volume to suppliers.
4. Threat from substitutes. Factors affecting substitute threat are relative price/performance of substitutes, buyer's
switching costs, and buyer's propensity to substitute.
5. The threat of new entry. Factors affecting entry barriers are capital requirements, access to distribution channels,
economies of scale, product differentiation, technological complexity of product or process, expected retaliation from
existing firms, and government policy.

We make three observations with regard to the industry analysis:

1. The more powerful the five forces are, the less profitable an industry is likely to be. In industries where average
profitability is high (such as soft drinks and pharmaceuticals), the five forces are weak (e.g., in the soft drink
industry, entry barriers are high). In industries where the average profitability is low (such as steel and coal), the five
forces are strong (e.g., in the steel industry, threat from substitutes is high).

2. Depending on the relative strength of the five forces, the key strategic issues facing the business unit will differ
from one industry to another.

48

3. Understanding the nature of each force helps the firm to formulate effective strategies. Supplier selection (a
strategic issue) is aided by the analysis of the relative power of several supplier groups; the business unit should link
with the supplier group for which it has the best competitive advantage. Similarly, analyzing the relative bargaining
power of several buyer groups will facilitate selection of target customer segments.

Generic Competitive Advantage:

The five-force analysis is the starting point for developing a competitive advantage since it helps to identify the
opportunities and threats in the external environment. With this understanding, Porter claims that the business unit
has two generic ways of responding to the opportunities in the external environment and developing a sustainable
competitive advantage: low cost and differentiation.

Low Cost: Cost leadership can be achieved through such approaches as economies of scale in production;

experience curve effects, tight cost control, and cost minimization (in such areas as research and development,
service, sales force, or advertising). Some firms following this strategy include Charles Schwab in discount
brokerage, Wal-Mart in discount retailing, Texas Instruments in consumer electronics, Emerson Electric in electric
motors, Hyundai in automobiles, Dell in computers, Black and Decker in machine tools, Nucor in steel, Lincoln
Electric in arc welding equipment, and BIC in pens.

Differentiation:

The primary focus of this strategy is to differentiate the product offering of the business unit, creating something that
is perceived by customers as being unique. Approaches to product differentiation include brand loyalty (Coca-Cola
and Pepsi Cola in soft drinks), superior customer service (Nordstrom in retailing), dealer network (Caterpillar
Tractors in construction equipment), product design and product features (Hewlett-Packard in electronics), and
technology (Cisco in communications infrastructure). Other examples of firms following a differentiation strategy
include BMW in automobiles; Stouffer's in frozen foods, Neiman-Marcus in retailing, Mont Blanc in pens, and Rolex
in wristwatches.

Value Chain Analysis:

Business units can develop competitive advantage based on low cost, differentiation, or both. The most attractive
competitive position is to achieve cost-cum-differentiation.

SET 8
1.

What is a responsibility centre? List and explain different types of Responsibility Centers with sketches.

Responsibility centers:
A responsibility center is an organization unit that is headed by a manager who is responsible for its activities. In a
sense, a company is a collection of responsibility centers. Each of which is represented by box on the on the
organization are responsibility centers for section work shifts or other small organization units. At a higher level are
departments or business units that consist of several of these smaller units plus staff and management people these
larger units are also responsibility center. And from the stand point of senior management and the board of directors,
the whole company is responsibility center although the term is usually used to refer to unit within the company.

49

Nature of responsibility centers


A responsibility center exist one or more purpose are its objectives. The company as a whole has goals, and senior
management has decided on a set of strategies to accomplish these goals. The objectives of responsibility centers are
to help implement these strategies. Because the organization is the sum of its responsibility centers, if the strategies
are sound and if each responsibility center, if the strategies are sound and if each responsibility center meets its
objectives the whole organization should achieve its goals. A responsibility center uses inputs, and a variety of
services. Its work with these resources and it usually require working capital, equipment, and other asset to do this
work. As a result of this work the responsibility center produces output which is classified either as goods if they are
tangible or as services if they are intangible. Every responsibility center has output that is it does something. In a
production plant, the outputs are goods. In staff units, such as human resources, transportation, engineering,
accounting, and administration, the output s are services. For many responsibility centers, especially staff units,
outputs are difficult to measure; nevertheless, they exist. The products produced by a responsibility center or to the
outside marketplace. In the first case, the product are inputs to the other responsibility center in the latter case, they
are output s of the whole organization.
Types of Responsibility Centers
Cost Center
Cost centers are divisions that add to the cost of the organization, but only indirectly add to the profit of the company.
Typical examples include Research and Development, Marketing and Customer service. Companies may choose to
classify business units as cost centers, profit centers, or investment centers. There are some significant advantages to
classifying simple, straightforward divisions as cost centers, since cost is easy to measure. However, cost centers
create incentives for managers to underfund their units in order to benefit themselves, and this underfunding may
result in adverse consequences for the company as a whole (reduced sales because of bad customer service
experiences, for example). Because the cost centre has a negative impact on profit (at least on the surface) it is a
likely target for rollbacks and layoffs when budgets are cut. Operational decisions in a contact centre, for example,
are typically driven by cost considerations. Financial investments in new equipment, technology and staff are often
difficult to justify to management because indirect profitability is hard to translate to bottom-line figures. Business
metrics are sometimes employed to quantify the benefits of a cost centre and relate costs and benefits to those of the
organization as a whole. In a contact centre, for example, metrics such as average handle time, service level and cost
per call are used in conjunction with other calculations to justify current or improved funding.

Profit Center
A responsibility centre is called a profit centre when the manager is held responsible for both costs (inputs) and
revenues (outputs) and thus for profit. Despite the name, a profit centre can exist in nonprofits organizations (though
it might not be referred to as such) when a responsibility centre receives revenues for its services. A profit centre is a

50

big segment of activity for which both revenues and costs are accumulated: A centre, whose performance is measured
in terms of both - the expense it incurs and revenue it earns, is termed as a profit centre. The output of a responsibility
centre may either be meant for internal consumption or for outside customers. In the latter case, the revenue is
realized when the sales are made. That is, when the output is meant for outsiders, then the revenue will be measured
from the price charged from customers. If the output is meant for other responsibility centre, then management takes
a decision whether to treat the centre as profit centre or not. In fact, any responsibility centre can be turned into a
profit centre by determining a selling price for its outputs. For instance, in case of a process industry, the output of
one process may be transferred to another process at a profit by taking into account the market price. Such transfers
will give some profit to that responsibility centre. Although such transfers do not increase the Companys assets, they
help in management control process.

Investment Centre
An investment centre goes a step further than a profit centre does. Its success is measured not only by its income but
also by relating that income to its invested capital, as in a ratio of income to the value of the capital employed. In
practice, the term investment centre is not widely used. Instead, the term profit centre is used indiscriminately to
describe centers that are always assigned responsibility for revenues and expenses, but may or may not be assigned
responsibility for the capital investment. It is defined as a responsibility centre in which inputs are measured in terms
of cost / expenses and outputs are measured in terms of revenues and in which assets employed are also measured. A
responsibility centre is called an investment centre, when its manager is responsible for costs and revenues as well as
for the investment in assets used by his centre. He is responsible for maintaining a satisfactory return on investment
i.e. asset employed in his responsibility centre. The investment centre manager has control over revenues, expenses
and the amounts invested in the centres assets. The manager of an investment centre is required to earn a satisfactory
return. Thus, return on investment (ROI) is used as the performance evaluation criterion in an investment centre. He
also formulates the credit policy, which has a direct influence on debt collection, and the inventory policy, which
determines the investment in inventory. The Vice President (Investments) of a mutual funds company may be in
charge of an Investment Centre. In the Investment Centre, the manager in charge is held responsible for the proper
utilization of assets. He is expected to earn a satisfactory return on the assets employed in his responsibility centre.
Measurement of assets employed poses many problems. It becomes difficult to determine the amount of assets
employed in a particular responsibility centre. Some of the assets are in the physical possession of the responsibility
centre while for some assets it may depend upon other responsibility centers or the Head Office of the company. This
is particularly true of cash or heavy plant and equipment. Whether such assets should be included in the figure of
assets employed of the responsibility centre and if included, at how much value, is a difficult question. On account of
these difficulties, investment centers are generally used only for relatively large units, which have independent
divisions, both manufacturing and marketing, for their individual products.

2.

Explain the process of evaluation of Responsibility Center from one stage to another with the help of
illustration-cum-experiences of the corporate.

Process of evaluation of Responsibility Center.

51

1.

The organization is divided into various responsibility centers. Each responsibility centre is put under the
charge of a responsibility manager.

2.

The targets or budgets of each responsibility centre are set in consultation with the manager of responsibility
centre, so that he may be able to give full information about his department. The manager of responsibility
centre should know as what is expected of him - each centre should have a clear set of goals. The
responsibility and authority of each centre should be well defined.

3.

Managers are charged with the items and responsibility, over which they can exercise a significant degree of
direct control.

4.

Goals defined for each area of responsibility should be attainable with efficient and effective performance.

5.

The actual performance is communicated to the managers concerned. If it falls short of the standards, the
variances are conveyed to the top management. The names of persons responsible for the variances are also
conveyed so that responsibility may be fixed.

The purpose of all these steps is to assign responsibility to different individuals so that their performance is improved
and costs are controlled. The personal factor in Responsibility Accounting is most important. The management may
prepare the best plan or the budget and put up before its staff, but its success depends upon the initiative and the will
of the workers to execute it
Example of Responsibility Center

The Sarva Shiksha Abhiyan emphasizes quality improvement in elementary education for which it deems necessary
that resource groups and responsibility centers from national to sub-district levels are identified. These groups would
oversee the policy, planning, implementation and monitoring of all quality related interventions. Their major role
would be to advise and assist at various levels in curriculum development, pedagogical improvement, teacher
education/training and activities related to classroom transaction. In order to facilitate a decentralized mode of
education, these groups would need to be constituted at various operational levels, namely - national, state, district
and sub district. The following could be involved in the groups:
National level - NCERT, NIEPA, Universities, NGOs, experts and eminent educationists.
State level - SCERT, SIEMAT, Universities, IASEs/CTEs, NGOs, experts and eminent educationists.
District level - DIETs, representatives from DPEP District Resource Group, higher educational institutions,
innovative teachers from the districts, NGOs.
Sub-district - BRC/BEO, representatives from CRCs, innovative teachers.
3.

Briefly define Discretionary Expense Center, Engineered Expense Center, Profit Centre and Investment
Centre? How is budget prepared in Discretionary Expenses Centre?

Engineered expense centers:


Engineered expense center have the following characteristics:
- Their inputs can be measured in monetary terms.
- Their output can be measured in physical terms.
- The optimal dollar amount of input required to produce one unit of output can be established.
Engineered expense center usually are found in manufacturing operations. Warehousing, distribution, trucking and
similar units in the marketing organization also may be engineered expense center and so many certain responsibility
center within administrative and support department. Examples are accounts receivable account payable and payroll
section in the controller department personnel record and cafeteria in the human resource department shareholder
record in the corporate secretary department and the company motor pool. Such units perform repetitive task for

52

which standard cost can be developed. In an engineered expense center the output multiplied by the standard cost of
each unit produced represents what the finished product should have cost. When this cost is compared to actual costs,
the difference between the two represents the efficiency of the organization unit being measured. We emphasize that
engineered expense centers have other important tasks not measured by cast alone. The effectiveness of this aspect of
performance should be controlled. For example expenses center supervisor are responsible for the quality of good
and for the volume of production in addition to their responsibility for cost efficiency. Therefore the type and amount
of production is prescribed and specific quality standards are set so that manufacturing costs are not minimized at the
expense of quality. Moreover manager of engineered expense center may be responsible for activities such a training
that are not related to current production judgment about their performance should include an appraisal of how well
they carry out these responsibilities. There are few if any responsibility center in which all cost items are engineered.
Even in highly automated production department the amount of indirect labor and of various services used can vary
with management discretion. Thus, the term engineered costs center refers to responsibility center in which
engineered cost predominate but in does not imply that valid engineering estimates can be made for each and every
cost item.
Discretionary expense center:
The output of discretionary expenses center cannot be measured in monitory terms. They include administration and
support units research and development organization and most marketing activities. The term discretionary does not
mean that management judgment is capricious or haphazard. Management has decided on certain policies that should
govern the operation of the company. Whether to match exceed or spend less than the marketing effort of its
competitor; the level of service that the company provides to the customer. The appropriate amount of spending for R
& D, financial planning public relation and many other activities. One company may have a small headquarter staff
another company of similar size and in the same industry may have a staff that is 10 times as large. the management
of both companies may be concerned that they made the correct decision on staff size but there is no objective way
judging which decision was actually better manager are hired and paid to make such decision. After such a drastic
change the level of discretionary expenses generally has a similar pattern from one year to the next. The difference
between budgeted and actual expense is not a measure of efficiency in a discretionary expense center it is simply the
difference between the budgeted input and the actual input. It in no way measures the value of the output. if actual
expense do not exceed the budget amount, the manager has lived within the budget however ,because by definition
the budget does not purport to measure the optimum amount of spending we cannot say that living within the
budgeted is efficient performance.
Profit Center
A responsibility centre is called a profit centre when the manager is held responsible for both costs (inputs) and
revenues (outputs) and thus for profit. Despite the name, a profit centre can exist in nonprofits organizations (though
it might not be referred to as such) when a responsibility centre receives revenues for its services. A profit centre is a
big segment of activity for which both revenues and costs are accumulated: A centre, whose performance is measured
in terms of both - the expense it incurs and revenue it earns, is termed as a profit centre. The output of a responsibility
centre may either be meant for internal consumption or for outside customers. In the latter case, the revenue is
realized when the sales are made. That is, when the output is meant for outsiders, then the revenue will be measured
from the price charged from customers. If the output is meant for other responsibility centre, then management takes
a decision whether to treat the centre as profit centre or not. In fact, any responsibility centre can be turned into a
profit centre by determining a selling price for its outputs. For instance, in case of a process industry, the output of
one process may be transferred to another process at a profit by taking into account the market price. Such transfers
will give some profit to that responsibility centre. Although such transfers do not increase the Companys assets, they
help in management control process.
Investment Centre
An investment centre goes a step further than a profit centre does. Its success is measured not only by its income but
also by relating that income to its invested capital, as in a ratio of income to the value of the capital employed. In
practice, the term investment centre is not widely used. Instead, the term profit centre is used indiscriminately to
describe centers that are always assigned responsibility for revenues and expenses, but may or may not be assigned
responsibility for the capital investment. It is defined as a responsibility centre in which inputs are measured in terms
of cost / expenses and outputs are measured in terms of revenues and in which assets employed are also measured. A
responsibility centre is called an investment centre, when its manager is responsible for costs and revenues as well as

53

for the investment in assets used by his centre. He is responsible for maintaining a satisfactory return on investment
i.e. asset employed in his responsibility centre. The investment centre manager has control over revenues, expenses
and the amounts invested in the centres assets. The manager of an investment centre is required to earn a satisfactory
return. Thus, return on investment (ROI) is used as the performance evaluation criterion in an investment centre. He
also formulates the credit policy, which has a direct influence on debt collection, and the inventory policy, which
determines the investment in inventory. The Vice President (Investments) of a mutual funds company may be in
charge of an Investment Centre. In the Investment Centre, the manager in charge is held responsible for the proper
utilization of assets. He is expected to earn a satisfactory return on the assets employed in his responsibility centre.
Budget Preparation.
The decision that management make about a discretionary expense budget are different from the decisions that it
makes about the budget for an engineered expense center. For the latter management decides whether the proposed
operating budget represent the cost of performing task efficiently for the coming period. management is not so much
concerned with the magnitude of the task because this is largely determined by the actions of other responsibility
centers, such as the marketing departments ability to generate sales. In formulating the budget for a discretionary
expense center, however management principal task is to decide on the magnitude of the job that should be done.
These tasks can be divided generally into two types continuing and special. Continuing task are those that continue
from year to year for example financial statement preparation by the controllers office. Special tasks are one shot
project for example developing and installing a profit budgeting system in a newly acquired division. The technique
management by objective is often used in preparing the budget for a discretionary expense center. Management by
objective is a formal process in which a budget purposes to accomplish specific tasks and state a mean for measuring
whether these tasks have been accomplished. There are two different approach to planning for the discretionary
expense center increment budgeting and zero based review.

SET .9
Q1. Describe inherent difficulties creation of profit centres may cause and advantages possible?
Under which situation creation of profit centre is not advisable.
Under which situation creation of profit centre is not advisable
Decentralized decision making will force top management to rely more on management control reports than on
personal knowledge of an operation, entailing some loss of control. If headquarters management is mere capable or
better informed than the average profit center manager, the quality of decisions made at the unit level way be
reduced. Friction may increase because of arguments over the appropriate transfer price, the assignment of common
costs, and the credit for revenues that were formerly generated jointly by two or more business units working
together.
Organization units that once cooperated as functional units may now be in competition with one another. An increase
in profits for one manager may mean a decrease for another. In such situation a manager may fail to refer sales leads
to another business unit better qualified to pursue them; may hoard personnel or equipment that, from the overall
companys, standpoint, would be better off used in another unit; or may make production decisions that have
undesirable cost consequences for other units.
Divisionalization may impose additional costs because of the additional management, staff personnel, and record
keeping required, and may lead to task redundancies at each profit center.Competent general managers may not exist
in a functional organization because there may not have been sufficient opportunities for them to develop general
management competence.
There may be too much emphasis on short-run profitability at the expense of long-run profitability. In the desire to
report high current profits, the profit center manager may skimp on R&D, training programs, or maintenance. This
tendency is especially prevalent when the turnover of profit center managers is relatively high. In these
circumstances, managers may have good reason to believe that their actions may not affect profitability until after
they have moved to other jobs. There is no completely satisfactory system for ensuring that optimizing the profits of
each individual profit center will optimize the profits of the company as a whole.
Q2.What are the challenges faced in pricing corporate services provided to Business Units
profit centers?
Business Units as Profit Centers

54

operating as

Most business units are created as profit centers since managers in charge of such units typically control product
development, manufacturing, and marketing resources. These managers are in a position to influence revenues and
costs and as such can be held accountable for the "bottom line." However, as pointed out in the next section, a
business unit manager's authority may be constrained in various ways, which ought to be reflected in a profit center's
design and operation.
Constraints on Business Unit Authority
To realize fully the benefits of the profit center concept, the business unit manager would have to be as autonomous
as the president of an independent company. As a practical matter, however, such autonomy is not feasible. If a
company were divided into completely independent units, the organization would lose the advantages of size and
synergy. Furthermore in delegating to business unit management all the authority that the board of directors has given
to the CEO, senior management would be abdicating its own responsibility. Consequently, business unit structures
represent trade-offs between business unit autonomy and corporate constraints. The effectiveness of a business unit
organization is largely dependent on how well these trade-offs are made.
Constraints from Other Business Units.
One of the main problems occurs when business units must deal with one another. It is useful to think of managing a
profit center in terms of control over three types of decisions:
(1) The product decision (what goods or services to make and sell),
(2) The marketing decision (how, where, and for how much are these goods or services to be sold?), and
(3) The procurement or sourcing decision (how to obtain or manufacture the goods or services). If a business unit
manager controls all three activities, there is usually no difficulty in assigning profit responsibility and measuring
performance. In general, the greater the degree of integration within a company,
the more difficult it becomes to assign responsibility to a single profit center for all three activities in a given product
line; that is, if the production, procurement, and marketing decisions for a single product line are split among two or
more business units, separating the contribution of each business unit to the overall success of the product line may
be difficult.
Constraints from Corporate Management
The constraints imposed by corporate management can be grouped into three types:
(1) Those resulting from strategic considerations,
(2) Those resulting because uniformity is required, and
(3) Those resulting from the economies of centralization.
Most companies retain certain decisions, especially financial decisions, at the corporate level, at least for domestic
activities. Consequently, one of the major constraints on business units results from corporate control over new
investments. Business units must compete with one another for a share of the available funds. Thus, a business unit
could find its expansion plans thwarted because another unit has convinced senior management that it has a more
Attractive program. Corporate management .also imposes other constraints. Each business unit has a "charter" that
specifies the marketing and/or production activities that it is permitted to undertake, and it must refrain from
operating beyond its charter, even though it sees profit opportunities in doing so. Also, the maintenance of the proper
corporate image may require constraints on the quality of products or on public relations activities.
Companies impose some constraints on business units because of the necessity for Uniformity. One-constraint is that
business Units must conform to corporate accounting and MCS This constraint is especially troublesome for units
that have been acquired from another company and that have been accustomed to using different systems.

Q.3) Write Short Notes on


1.

Zero Based Budgeting

2.

Internal Control\

Zero Based Budgeting:


Zero-based budgeting is a technique of planning and decision-making which reverses the working process of
traditional budgeting. In traditional incremental budgeting, departmental managers justify only increases over the
previous year budget and what has been already spent is automatically sanctioned. No reference is made to the

55

previous level of expenditure. By contrast, in zero-based budgeting, every department function is reviewed
comprehensively and all expenditures must be approved, rather than only increases. [1] Zero-based budgeting requires
the budget request be justified in complete detail by each division manager starting from the zero-base. The zero-base
is indifferent to whether the total budget is increasing or decreasing.
The term "zero-based budgeting" is sometimes used in personal finance to describe the practice of budgeting
every dollar of income received, and then adjusting some part of the budget downward for every other part that needs
to be adjusted upward. It would be more technically correct to refer to this practice as "active-balanced budgeting".
Advantages of Zero-Based Budgeting:
1.
2.
3.
4.
5.
6.
7.
8.
9.
10.

Efficient allocation of resources, as it is based on needs and benefits.


Drives managers to find cost effective ways to improve operations.
Detects inflated budgets.
Municipal planning departments are exempt from this budgeting practice.
Useful for service departments where the output is difficult to identify.
Increases staff motivation by providing greater initiative and responsibility in decision-making.
Increases communication and coordination within the organization.
Identifies and eliminates wasteful and obsolete operations.
Identifies opportunities for outsourcing.
Forces cost centers to identify their mission and their relationship to overall goals.

Disadvantages of Zero-Based Budgeting:


1.
2.
3.

4.
5.

Difficult to define decision units and decision packages, as it is time-consuming and exhaustive.
Forced to justify every detail related to expenditure. The R&D department is threatened whereas the
production department benefits.
Necessary to train managers. Zero-based budgeting must be clearly understood by managers at various
levels to be successfully implemented. Difficult to administer and communicate the budgeting because more
managers are involved in the process.
In a large organization, the volume of forms may be so large that no one person could read it all.
Compressing the information down to a usable size might remove critically important details.
Honesty of the managers must be reliable and uniform. Any manager that exaggerates skews the results

Internal Control:
Internal control is defined as a process affected by an organization's structure, work and authority flows,
people and management information systems, designed to help the organization accomplish specific goals or
objectives.[1] It is a means by which an organization's resources are directed, monitored, and measured. It plays an
important role in preventing and detecting fraud and protecting the organization's resources, both physical (e.g.,
machinery and property) and intangible (e.g., reputation or intellectual property such as trademarks). At the
organizational level, internal control objectives relate to the reliability of financial reporting, timely feedback on the
achievement of operational or strategic goals, and compliance with laws and regulations. At the specific transaction
level, internal control refers to the actions taken to achieve a specific objective (e.g., how to ensure the
organization's payments to third parties are for valid services rendered.) Internal control procedures reduce process
variation, leading to more predictable outcomes
Describing Internal Controls:
Internal controls may be described in terms of: a) the objective they pertain to; and b) the nature of the control
activity itself.
Objective categorization
Internal control activities are designed to provide reasonable assurance that particular objectives are achieved, or
related progress understood. The specific target used to determine whether a control is operating effectively is called
the control objective. Control objectives fall under several detailed categories; in financial auditing, they relate to
particular financial statement assertions,[5] but broader frameworks are helpful to also capture operational and
compliance aspects:

56

1.
2.
3.
4.
5.
6.
7.

Existence (Validity): Only valid or authorized transactions are processed (i.e., no invalid transactions)
Occurrence (Cutoff): Transactions occurred during the correct period or were processed timely.
Completeness: All transactions are processed that should be (i.e., no omissions)
Valuation: Transactions are calculated using an appropriate methodology or are computationally accurate.
Rights & Obligations: Assets represent the rights of the company, and liabilities its obligations, as of a given
date.
Presentation & Disclosure (Classification): Components of financial statements (or other reporting) are
properly classified (by type or account) and described.
Reasonableness-transactions or results appear reasonable relative to other data or trends.

Activity categorization
Control activities may also be described by the type or nature of activity. These include (but are not limited to):

Segregation of duties - separating authorization, custody, and record keeping roles to limit risk of fraud or
error by one person.
Authorization of transactions - review of particular transactions by an appropriate person.
Retention of records - maintaining documentation to substantiate transactions.
Supervision or monitoring of operations - observation or review of ongoing operational activity.
Physical safeguards - usage of cameras, locks, physical barriers, etc. to protect property.
Analysis of results, periodic and regular operational reviews, metrics, and other key performance indicators
(KPIs).
IT Security - usage of passwords, access logs, etc. to ensure access restricted to authorized personnel. S

Q4 .Veena Pvt. Ltd., a small multiproduct company is taken over by a multinational company ( e.g. Hindustan
Lever.) What changes in the control system would you expect and why?
Since Veena is a small multiproduct company it would require changes in control system which would be related to
transfer pricing a, as this company would generally provide inputs to HUL. Thus the domestic operations generally
involve transfer of goods and services only In view of this difference many other considerations, in addition to the
criteria used in domestic operations for the determination of transfer price, are involved. These include:
(a) Fair Price: This is an important factor one needs to consider while determining the transfer price for foreign
operations. Companies that enter into joint ventures must ensure that the transfer price charged is fair. If such
companies charge a higher transfer price, it would reduce the profits of the joint venture and as a result reduce the
foreign partner's share of profits.
(b) Government Regulations: All countries have a regulatory framework under which business units operate. Where
government rules and regulations regarding transfer prices are lenient, the parent company should fix a higher
transfer price for all transfers to countries with high income tax rates. This approach would enable the parent
company to minimize taxes in such countries.
(c) Exchange Control Restrictions: Every country has foreign exchange control regulations.
These regulations impose a limit on the amount of foreign exchange available for the import of certain goods. To
accommodate the foreign subsidiary the parent company may have a lower transfer price so that the subsidiary is able
to import a larger quantity of required goods.
(d) Income Tax Regulations: The rates of income tax vary from country to country. To overcome this difference the
transfer price should be so fixed that countries with low tax rates show profits while others end up with a loss. This
helps the parent company to reduce its taxes on a global basis.
(e) Desire to accumulate funds: A company that wishes to accumulate funds in a particular country may fix the
transfer prices in such a manner that it facilitates shifting of funds into that country.
(I) Tariffs- and Duties: No country likes high imports. In order to restrict imports countries impose restrictions such
as quantitative restrictions, high duties and tariffs and banning import of products. The general practice is to charge
import duties as a percentage of the value of products imported, although a lower tariff may be levied if the import
value is lower. It is seen that the impact of tariffs on the profitability of foreign operations is generally the reverse of
the incidence of income taxes in transfer pricing. As such a low transfer price would lead to low import duties on

57

transfer, the profit arising in that country would be high. This results in high income taxes in that country. It is
therefore advisable that companies must compute the net effect of these factors while determining transfer prices.
In designing performance evaluation systems for acquired Veena company,HUL could use the following guidelines
Subsidiary managers should not be held responsible for translation effects. The simplest way to achieve this objective
is to compare budgets and actual results using the same metric and isolate inflation-related effects through variance
analysis. It is pointless for managers to worry about the appropriate metric. The MNE should choose whatever metric
is more convenient.
Transaction effects are best handled through centralized coordination of the MNE's overall hedging needs. This is
likely to be cheaper and simpler, and it prevents the subsidiary manager from becoming a foreign exchange rate
forecaster and speculator.
The subsidiary manager should be held responsible for the dependence effects of exchange rates resulting from
economic exposure.
Evaluation of the subsidiary as a basis for a decision to locate operations in a country or to relocate operations from a
country should reflect the consequences of translation. Transaction and economic exposures.

SET 10
Q.1) What are the Special Characteristics of Professional Service Organization?
Answer:
Goals
A dominant goal of a manufacturing company is to earn a satisfactory profit, specifically a satisfactory return on
assets employed. A professional organization has relatively few tangible assets; its principal asset is the skill of its
professional staff, which 'doesn't appear on its balance sheet. Return on assets employed, therefore, is essentially
meaningless in such organizations. Their financial goal is to provide adequate compensation to the professionals.
In many organizations, a related goal is to increase their size. In part, this reflects the natural tendency to associate
success with large size. In part, it reflects economies of scale in using the efforts of a central personnel staff and units
responsible for keeping the organization up-to-date. Large public accounting firms need to have enough local offices
to enable them to audit clients who have facilities located throughout the world.
Professionals
Professional organizations are labor intensive, and the labor is of a special type. Many professionals prefer to work
independently, rather than as part of a team. Professionals who are also managers tend to work only part time on
management activities; senior partners in an accounting firm participate actively in audit engagements; senior
partners in law firms have clients. Education for most professions does not include education in management, but
quite naturally stresses the skills of the profession, rather than management; for this and other reasons, professionals
tend to look down on managers.
Professionals tend to give inadequate weight to the financial implications of their decisions; they want to do the best
job they can, regardless of its cost. This attitude affects the attitude of support staffs and nonprofessionals in the
organization; it leads to inadequate cost control.
Output and Input Measurement
The output of a professional organization cannot be measured in physical terms, such as units, tons, or gallons. We
can measure the number of hours a lawyer spends on a case, out this is a measure of input, not output. Output is the
effectiveness of the lawyer's work, and this is not measured by the number of pages in a brief or the number of hours
in the courtroom. We can measure the number of patients a physician Teats in a day, and even classify these visits by
type of complaint; but this is by no means equivalent to measuring the amount or quality of service the physician has
provided. At most, what is measured is tl1e physician's efficiency in treating patients, which is of some use in
identifying slackers and hard workers. Revenues earned is one measure of output in some professional organizations,
but these monetary amounts, at most, relate to the quantity of services rendered, not to their quality (although poor
quality is reflected in reduced revenues in the long run).

58

Furthermore, the work done by many professionals is non repetitive. No two consulting jobs or research and
development projects are quite the same. This makes it difficult to plan the time required for a task, to set reasonable
standards for task performance, and to judge how satisfactory the performance was. Some tasks are essentially
repetitive: the drafting of simple wills, deeds, sales contracts, and similar documents; the taking of a physical
inventory by an auditor; and certain medical and surgical procedures. The development of standards for such tasks
may be worthwhile, although in using these standards, usual circumstances that affect a specific job must be taken
into account. Some professionals, notably scientists, engineers, and professors, are reluctant to keep track of how
they spend their time, and this complicates the task of measuring performance. This reluctance seems to have its roots
in tradition; usually, it can be overcome if senior management is willing to put appropriate emphasis on the necessity
of accurate time reporting. Nevertheless, difficult problems arise in deciding how time should be charged to clients.
If the normal work week is 40 hours, should a job be charged for 1140th of a week's compensation for each hour
spent on it? If so, how should work done on evenings and weekends be counted? (Professionals are "exempt"
employees-that is, they are not subject to government requirements of overtime payments.) How to account for time
spent reading literature, going to meetings, and otherwise keeping up-to-date?
Small Size
With a few exceptions, such as some law firms and accounting firms, professional organizations are relatively small
and operate at a single location. Senior management in such organization can personally observe what is going on
and personally motivate employees. Thus, there is less need for a sophisticated management control system, with
profit centers and formal performance reports. Nevertheless, even a small organization needs a budget, a regular
comparison of performance against budget, and a way of relating compensation to performance.
How is Marketing done in them?
In a manufacturing company there is a clear dividing line between marketing activities and production activities; only
senior management is concerned with both. Such a clean separation does not exist in most professional organizations.
In some, such as law, medicine, and accounting, the profession's ethical code limits the amount and character of overt
marketing efforts by professionals (although these restrictions have been relaxed in recent years). Marketing is an
essential activity in almost all organizations, however. If it can't be conducted openly, it takes the form of personal
contacts, speeches, articles, conversations on the golf course, and so on. These marketing activities are conducted by
professionals, usually by professionals who spend much of their time in production work-that is, working for clients.
In this situation, it is difficult to assign appropriate credit to the person responsible for "selling" a new customer. In a
consulting firm, for example, a new engagement may result from a conversation between a member of the firm and
an acquaintance in a company, or from the reputation of one of the professionals as an outgrowth of speeches or
articles.
Moreover, the professional who is responsible for obtaining the engagement may not be personally involved in
carrying it out. Until fairly recently, these marketing contributions were rewarded subjectively-that is, they were
taken into account in promotion and compensation decisions. Some organizations now give explicit credit, perhaps
as a percentage of the project's revenue, if the person who "sold" the project can be identified.
How do we evaluate the Performance Appraisal?
As noted earlier in regard to teachers, at the extremes the performance of professionals is easy to judge. Appraisal of
the large percentage of professionals who are within the extremes is much more difficult. For some professions,
objective measures of performance are sometimes unavailable: The recommendations of an investment analyst can be
compared with actual market behavior of the securities; the accuracy of a surgeon's diagnosis can be verified by an
examination of the tissue that was removed; and the doctors' skill can be measured by the success ratio of operations.
These measures are, of course, subject to appropriate qualifications, and in most circumstances the assessment of
performance is finally a matter of human judgment by superiors, peers, self, subordinates, and clients. Judgments
made by superiors are the most common. For these, professional organizations increasingly use formal systems to
collect performance appraisals as a basis for personnel decisions and for discussion with the professional. Some
systems require numerical ratings of specified attributes of performance and provide for a weighted average of these
ratings. Compensation may be tied, in part, to these numerical ratings. In a matrix organization, both the project
leader and the head of the functional unit that is the professional's organizational "home" judge performance. "
Appraisals by a professional's peers, or by subordinates, are sometimes part of a formal control system. In some
organizations, individuals may be asked to make a self-appraisal. Expressions of satisfaction or dissatisfaction from
clients are also an important basis for judging performance, although such expressions may not always be readily
forthcoming.
The budget can be used as the basis for measuring cost performance, and the actual time taken can be compared with
the planned time. Budgeting and control of discretionary expenses are as important in a professional firm as in a
manufacturing company.

59

Such financial measures are relatively unimportant in assessing a professional's contribution to the firm's,
profitability, however. The professional's major contribution is related to quantity and above all quality of work, and
its appraisal must be largely subjective. Furthermore, the appraisal must be made currently; it cannot wait until one
learns whether a new building is well designed, a new control system actually works well, or a bond indenture has a
flaw.
In some professions, internal audit procedures are used to control quality. In many accounting firms, the report of an
audit is reviewed by a partner other than the one who is responsible for it, and the work of the whole firm is "peer
reviewed" by another firm. The proposed design of a building may be reviewed by architects who are not actively
involved in the project.
Q.2) What is a Non - Profit Organization? How is the performance of this organization evaluated?
Answer:
Introduction
A nonprofit organization, as defined by law, is an organization that cannot distribute assets or income to, or for the
benefit of, its members, officers, or directors. The organization can, of course, compensate its employees, including
officers and members, for services rendered and for goods supplied. This definition does not prohibit an organization
from earning a profit; it prohibits only the distribution of profits. A nonprofit organization needs to earn a modest
profit, on average, to provide funds for working capital and for possible rainy days.
Performance evaluation of nonprofit organization
For any organization, the most important reasons to measure performance are to improve effectiveness and to acquire
information that will allow the organization to drive its agenda forward. If the motivation for doing evaluation
remains outside an organization, the evaluation will have limited impact. To do performance assessment effectively,
an organization must commit to adopting a culture of measurement, because acceptance must come from senior
management, staff, funders, and board members alike.

Board self-evaluation
Members of the Board of Directors should regularly evaluate the quality of their activities on a regular basis.
Activities might include staffing the Board with new members, developing the members into well-trained and
resourced members, discussing and debating topics to make wise decisions, and supervising the CEO. Probably the
biggest problem with Board self-evaluation is that it does not occur frequently enough. As a result, Board members
have no clear impression of how they are performing as members of a governing Board. Poor Board operations, when
undetected, can adversely affect the entire organization.
Staff and volunteer (individual) performance evaluation
Most of us are familiar with employee performance appraisals, which evaluate the quality of an individuals
performance in their position in the organization. Ideally, those appraisals reference the individuals written job
description and performance goals to assess the quality of the individuals progress toward achieving the desired
results described in those documents. Continued problems in individual performance often are the results of poor
strategic planning, program planning and staff development. If overall planning is not done effectively, individuals
can experience continued frustration, stress and low morale, resulting in their poor overall performance. Experienced
leaders have learned that continued problems in performance are not always the result of a poor work ethic the
recurring problems may be the result of larger, more systemic problems in the organizations.
Program evaluation
Program evaluations have become much more common, particularly because donors demand them to ensure that their
investments are making a difference in their communities. Program evaluations are typically focused on the quality
of the programs process, goals or outcomes. An ineffective program evaluation process often is the result of poor
program planning programs should be designed so they can be evaluated. It can also be the result of improper
training about evaluation. Sometimes, leaders do not realize that they have the responsibility to verify to the public
that the nonprofit is indeed making a positive impact in the community. When program evaluations are not performed
well, or at all, there is little feedback to the strategic and program planning activities. When strategic and program
planning are done poorly, the entire organization is adversely effected.
Evaluation of cross-functional processes
Cross-functional processes are those that span several systems, such as programs, functions and projects. Common
examples of major processes include information technology systems and quality management of services. Because
these cross-functional processes span so many areas of the organization, problems in these processes can be the result
of any type of ineffective planning, development and operating activities.
Organizational evaluation

60

Ongoing evaluation of the entire organization is a major responsibility of all leaders in the organization. Leaders
sometimes do not recognize the ongoing activities of management to actually include organizational evaluations
but they do. The activities of organizational evaluation occur every day. However, those evaluations usually are not
done systematically. As a result, useful evaluation information is not provided to the strategic and program planning
processes. Consequently, both processes can be ineffective because they do not focus on improving the quality of
operations in the workplace.
Q.3) A Well Diversified company Pritam International Ltd. sells one of its divisions to a group of its own company
managers. Explain what significant changes in systems and control procedures can be expected? Why?
Answer:
As, we Pritam International is a well diversified company. Sometimes, excessive diversification and that too in
unrelated lines of business causes failure in the business operations. One of the major reason for failures of many
Mergers and Diversification is excessive diversification. As, excessive diversification is ominous especially, in
unrelated lines of business. As, there may be no advantage of operating synergy. Neither through:
I) Sharing common resources nor
II) Sharing common core competencies
Therefore, it may be a strategic decision by the promoters and directors of the company to sell one of its divisions.
As, this may be impacting their core business. Sometimes, your core business tends to get neglected mainily due to
excesive diversification. As, the division is being sold to its own company managers. There, might not be major
changes in management control and systems. As, most of its managers will be the same. But, they will have more
autonomy to take decisions independently after acquisition. Now there will be less red tapism and managers can take
more risk. The managers will manage the firm in their own style. As, they are not answerable to their superiors.
Currently, they are answerable to their stake-holders. As, the management is completely in their hands and that too
with full autonomy. The management might have identified the flaws in the previous controls and systems of the
company because of which the company might not be so effective and efficient. As, they have been associated with
the company over aperiod of time. They have a better understanding about the business dynamics and environment in
which the firm operates. So, they can take necessary steps to overcome the flaws and improve the management
control and systems.
So, that is why there will be some significant changes in the management control and systems and procedures if there
is further scope for improvement.

SET 11
Q.1)Why Balance Score Card is considered superior to other methods of Performance Appraisal?
Prepare Balance Score Card for any organization you are familiar with.
ANSWER:
What is the Balanced Scorecard?
The rationale for the development of the Balanced Scorecard was a growing dissatisfaction with
traditional, financial measures of performance. These measures suffer from a number of serious drawbacks
in that they take a short-term, lagged (i.e., historic) view of performance. The shift towards flexible, lean
production/service systems in many firms has strengthened the requirement for performance measurement
systems to become more broadly based, incorporating both non-financial and external measures of
performance. According to Kaplan and Norton, the Balanced Scorecard provides a better assessment of
performance as it "enables companies to track financial results while simultaneously monitoring progress
in building the capabilities and acquiring the intangible assets they need for future growth".
The original scorecard designed by Kaplan and Norton contained four key groupings of performance
measures. These four groupings, called perspectives by Kaplan and Norton, were considered sufficient to
track the key drivers of both current and future financial performance of the firm. The perspectives
focused on the achievements of the firm in four areas: namely the financial, customer, internal business
process and innovation/learning perspectives. The four perspectives can be represented as an interlinked
61

hierarchy. The firms strategy underlies the whole scorecard, as the measures for each of the four
perspectives are drawn from this strategy.

To obtain a satisfactory overview of performance, the scorecard will require a mix of lagging and leading
(forward looking) measures. Financial measures tend to be lagged and consequently, the measures chosen
for the other perspectives will need to include leading measures. In general, outcome measures tend to be
lagged, for example, current market share is the result of past decisions and consequently is a lagging
measure. Thus the challenge in designing a Balanced Scorecard is to choose driver measures which lead
changes in the outcome measures in the non-financial perspectives and which ultimately drive the
financial measures.
Once the firms objectives have been agreed and the appropriate outcome and driver measures chosen for
each of the perspectives, firm and managerial performance is assessed by comparing actual attainment on
each measure with the target set for that measure.
Objective

Measure

Target

Actual

Benefits from adopting the Balanced Scorecard


There are several benefits from implementing a Balanced Scorecard. Originally the Balanced Scorecard
was seen as a useful tool for performance measurement. In this role, the Balanced Scorecard was seen as
integrating financial/non-financial, internal/external and leading /lagging information on firm performance
in a coherent fashion.
Later it was realised that the Balanced Scorecard could play a pivotal role in the strategic management
process. Because the Balanced Scorecard requires management to clarify and obtain consensus on the
strategic objectives of the firm, it can assist in the communication of the chosen strategy, consequently
aligning the efforts both of individuals and of departments. In this role, there is a clear link between the
Balanced Scorecard and management by objectives (MBO). Effective implementation of a Balanced
Scorecard project will generally involve the development of a series of hierarchical (cascaded) scorecards.
Given the overall corporate scorecard, supporting scorecards can be developed for each department within
the firm. Within each department, a scorecard can be developed for each manager (or perhaps even for
each individual member of staff) which links the objectives on each perspective for that manager back to
62

the objectives for each perspective outlined in the scorecard for the department and finally, back to the
objectives listed in the firms overall scorecard.
The Balanced Scorecard could be used to assist in corporate restructuring. In recent years, many firms
have migrated away from a traditional hierarchical structure to a flatter, team-based organisational
structure. The Balanced Scorecard can support such changes, as it can help clarify the objectives and the
critical success factors for the newly formed teams.
Apart from the communication and co-ordination roles of the Balanced Scorecard in strategic
implementation, the Balanced Scorecard can be used to link strategy to specific critical success factors in
the customer, internal business process and growth/learning perspectives. By setting both short and longterm targets for driver and outcome measures and by comparing actual attainment against target, feedback
is obtained on how well the strategy is being implemented and on whether the strategy is working.
Building on the Balanced Scorecards use as a strategic management tool, it has been suggested that the
Balanced Scorecard can play a role in the investment appraisal process(5). Traditional methods of
investment appraisal such as discounted cash flow do not cope well with investments which generate
indirect rather than direct financial returns. Examples of these include investments which enhance the
future flexibility of a firm or investments in the firms infrastructure, such as an enhanced management
information system. The Balanced Scorecard can assist managements investment appraisal decisions as it
provides managers with a mechanism to incorporate the strategic aspects of the investment into the
appraisal process. This could be achieved by using a weighting system developed from a firms Balanced
Scorecard measures to evaluate new projects. An index score would be calculated for each investment
opportunity and projects would then be ranked and selected based on this score.
Balance Score Card of Credit Card Company

63

Q .2)
Soniya Company has two Divisions: A & B. Return on Investment for both divisions is 20%. Details
are given below:Particulars

Div A

Div B

Divisional sales

4000000

9600000

Divisional Investment

2000000

3200000

Profit

400000

640000

Analyse and comment on divisional performance of each.


ANSWER
As Profit Margin = Profit *100
Sales
Profit Margin for Division A= 4,00,000 /40,00,000 *100 = 10%
Profit Margin for Division B = 6,40,000/ 96,00,000 *100 = 6.6%
Turnover of Investment = Sales * 100
Investment

Turnover of Investment for Division A = 40,00,000/20,00,000 = 2 times


Turnover of Investment for Division B = 96,00,000/32,00,000 = 3 times
As Return on investment for both Divisions A and B is 20%.
COMMENTS:Division A Although A has more profit margin than Division B that is 10% as compared to 6.6% of
B, so it has more profitability but inspite of it, division A has lower turnover of investment that its
assets management is bad than Division B, it can be improved by increased sales or reducing investment.
Division B Needs to improve profit margin by increasing sales and reduce variable cost and sales at
same price or by reducing salesprice and increase the volume of sales so that its profit would improve. As
it has good assets management shown by its turnoverof Division B that is 3 times which is better than
Division A. So it can become profitable organisation by improving Profit Margin.
Q.4)Discuss and illustrate differences and similarities between
- Strategy Formulation and Management Control
- Management Control and Task Control

ANSWER
64

Some Distinction between Strategy Formulation and management Control

Characteristics

Strategy Formulation

Management Control

a) Focus of plan

On one aspect at a time

On entire organisation

b) Complexities

Many variables hence complex

Less complex

c) Nature of information

Tailor-made for the issue, more Integrated, more internal and


external and predictive, less historical, more accurate.
accurate.

d) Structure

Unstructured and irregular, each Rhythmic, definite pattern, set


problem being different
procedure

e)
Communication
information

of Relatively simple

Relatively difficult

f) Purpose of estimates

Show expected results

Lead to desired result

g) Persons involved

Staff and top management

Line and top management

h) No. of persons involved

Small

Large

i) Mental activity

Creative, analytical

Administrative, persuasive

j) Planning and control

Planning dominant but some Emphasis on both planning and


control
control

k) Time horizon

Tends to be long

Tends to be short

l) End result

Policies and precedents

Action within policies laid

m) Appraisal of job done

Extremely difficult

Less difficult

65

b) Some Distinction between Management Control and Task Control


Characteristics

Task Control

Management control

a) Focus of plan

Single task or transaction

On entire organisation

b) Nature of information

Tailor-made
to
operation, Integrated, more internal and
specific, often non- financial, historical, more accurate
real time

c) Persons involved

Supervisors

d) Mental activity

Follow directives or none as in Administrative, persuasive


case of machines or set
objectives

e) Time horizon

Day to day

f) Type of cost

EngineeredExistence
of Discretionary- Control is more
objective
standard
against difficult due to subjective
which actuals can be compared consideration.
makes control easier.

Line and top management

Tends to be short

SET 12
Q.2 Suresh Ltd. (Numerical) (MCS-2007)

(a) Define profit in this case and prepare a statement for both divisions and overall company.
Solution:
i) Profitability statement of Division A:Particulars
Selling price p.u.
Variable Cost p.u.
Contribution p.u.

Contribution p.u.
24
24
24

Amount(Rs.)
35
11
24

Expected
sales Total contribution
(no. of units)
2000
48000
3000
72000
6000
144000

66

Total Fixed cost Net profit (Rs.)


(Rs.)
60000
(12000)
60000
12000
60000
84000

ii) Profitability statement of Division B:Selling p.u.

Total
Contribution
variable cost p.u.
p.u.
90
42
48
80
42
38
50
42
8
[Note: Total Variable cost p.u. = Variable
(Rs.35)]

Expected
Total
Total Fixed Net
profit
sales (no. of contribution cost (Rs.)
(Rs.)
units)
2000
96000
90000
6000
3000
114000
90000
24000
6000
48000
90000
(42000)
cost p.u. (Rs.7) + Transfer price of intermediate product

iii) Profitability statement of Company as a whole:Expected sales


2000
3000
6000

Net profit of division A Net profit of Division B Total Net profit


(Rs.)
(Rs.)
(12000)
6000
(6000)
12000
24000
36000
84000
(42000)
42000

(b) State the selling price which maximizes profits for division B and company as a whole. Comment
on why the latter price is unlikely to be selected by division B.
Solution:
As per the calculation in part (a), selling price p.u. of Rs.80 maximizes profit for division B whereas
selling price p.u. of Rs.50 maximizes profit for the Company as a whole. However, if Division B opts
for selling price p.u. of Rs.50 in order to maximize Companys profit, it would suffer a loss of
Rs.42000. Therefore, Division B would not select Selling price p.u. of Rs.50.

67

68

Q.3 Explain different organizational goals. Comment on goal of shareholder value maximization
in particular.
Goals
Although we often refer to the goals of a corporation, a corporation does not have goals; it is an artificial
being with no mind or decision-making ability of its own. Corporate goals are determined by the chief
executive officer (CEO) of the corporation, with the advice of other members of senior management, and
they are usually ratified by the board of directors. In many corporations, the goals originally set by the
founder persist for generations. Examples are Henry Ford, Ford Motor Company; Alfred P. Sloan, General
Motors Corporation; Walt Disney, Walt Disney Company; George Eastman, Eastman Kodak; and Sam
Walton, Wal-Mart.
Economic Goals
Shareholder's value, Earning per share and Market value, all relate to maximizing shareholder's value,
which is not a desirable goal, because what is 'maximum' is difficult to determine. Although optimizing
shareholder value may be one goal, but there are other stakeholders in the business also such as customers,
employees, creditors, community and so on. Again, shareholder value is usually equated with the market
value of the company's stock. But market value is not an accurate measure of the worth of shareholders'
investments. Besides, such value can be obtained only when the share is traded in the stock exchange.
It is interesting to note that Henry Ford's operating philosophy was 'satisfactory profit', not 'maximum
profit'. He said, "A reasonable profit is right, but not too much. So, it has been my policy to force the price
of the car down as fast as production would permit and give the benefit to the user and laborers, with
resulting surprisingly enormous benefit to ourselves"
Other goals such as adding new products, or product-line or new business actually indicate normal
organizational growth.
Social Goals
However, every organization has its share of responsibility towards the local community where it is
situated, and the public at large. It is very difficult to incorporate in Management Control System
such goals as taking pride in an organization which cares for the society and renders service to the
public. Of course, any concrete structural programme indicating its operational expenses, methods of
providing service, personnel involved in rendering service and the nature of the service in details can,
however, be mentioned through an appropriate system.
Profitability
In a business, profitability is usually the most important goal.
Return on investment can be found by simply dividing profit (i.e., revenues minus expenses) by investment, but this method does not draw attention to the two principal components: profit margin and
investment turnover.
In the basic form of this equation, "investment" refers to the shareholders' investment, which consists
of proceeds from the issuance of stock, plus retained earnings.
One of management's responsibilities is to arrive at the right balance between the two main sources of
financing: debt and equity. The shareholders' investment (i.e., equity) is the amount of financing that was
not obtained by debt, that is, by borrowing. For many purposes, the source of financing is not relevant;
"investment" thus means the total of debt capital and equity capital.
"Profitability" refers to profits in the long run, rather than in the current quarter or year. Many current
expenditure (e.g., amounts spent on advertising or research and development) reduce current profits but
increase profits over time.
Some CEOs stress only part of the profitability equation. Jack Welch, former CEO of General Electric
Company, explicitly focused on revenue; he stated that General Electric should not be in any business in
69

which its sales revenues were not the largest or the second largest of any company in that business. This
does not imply that Welch neglected the other components
of the equation; rather, it suggests that in his mind there was a close correlation between market share
and return on investment.
Other CEOs, however, emphasize revenues for a different reason: For them, company size is a goal.
Such a priority can lead to problems. If expenses are too high, the profit margin will not give
shareholders a good return on their investment. Even if the profit margin is satisfactory, the organization
may still not earn a good return if the investment is too large.
Some CEOs focus on profit either as a monetary amount or as a percentage of revenue. This focus
does not recognize the simple fact that if additional profits are obtained by a greater than proportional
increase in investment, each dollar of investment has earned less.
Maximizing Shareholder Value
In the 1980s and 1990s the term shareholder value appeared frequently in the business literature. This
concept is that the appropriate goal of a for-profit corporation is to maximize shareholder value. Although
the meaning of this term was not always clear, it probably refers to the market price of the corporation's
stock. We believe, however, that achieving satisfactory profit is a better way of stating a corporation's
goal, for two reasons.
First, "maximizing" implies that there is a way of finding the maximum amount that a company can
earn. This is not the case. In deciding between two courses of action, management usually selects the one
it believes will increase profitability the most. But management rarely, if ever, identifies all the possible
alternatives and their respective effects on profitability. Furthermore, profit maximization requires that
marginal costs and a demand curve be calculated, and managers usually do not know what these are. If
maximization were the goal, managers would spend every working hour (and many sleepless nights)
thinking about endless alternatives for increasing profitability; life is generally considered to be too short
to warrant such an effort.
Second, although optimizing shareholder value may be a major goal, it is by no means the only goal for
most organizations. Certainly a business that does not earn a profit at least equal to its cost of capital is not
doing its job; unless it does so, it cannot discharge any other responsibilities. But economic performance
is not the sole responsibility of a business, nor is shareholder value. Most managers want to behave
ethically, and most feel an obligation to other stakeholders in the organization in addition to shareholders.
Example: Henry Ford's operating philosophy was satisfactory profit, not maximum profit. He wrote
let me say right here that I do not believe that we should make such an awful profit on our cars. A
reasonable profit is right, but not too much. So it has been my policy to force the price of the car
down as fast as production would permit, and give the benefits to the users and laborers-with
resulting surprisingly enormous benefits to ourselves.
By rejecting the maximization concept, we do not mean to question the validity of certain obvious
principles. A course of action that decreases expenses without affecting another element, such as market
share, is sound. So is a course of action that increases expenses with a greater than proportional increase in
revenues, such as expanding the advertising budget. So, too, are actions that increase profit with a less than
proportional increase in shareholder investment (or, of course, with no such increase at all), such as
purchasing a cost-saving machine. These principles assume, in all cases, that the course of action is ethical
and consistent with the corporation's other goals.
An organization's pursuit of profitability is affected by management's willingness to take risks. The degree
of risk-taking varies with the personalities of individual managers. Nevertheless there is always an upper
limit; some organizations explicitly state that management's primary responsibility is to preserve the
company's assets, with profitability considered a secondary goal. The Asian .financial crisis during 19961998 is traceable, in large part, to the fact that banks in Asia's emerging markets made what appeared to be
highly profitable loans without paying adequate attention to the level of risk involved.

70

Multiple Stakeholder Approach


Organizations participate in three markets: the capital market, the product market, and the factor market. A
firm raises funds in the capital market, and the public stockholders are therefore an important constituency.
The firm sells its goods and services in the product market, and customers form a key constituency. It
competes for resources such as human capital and raw materials in the factor market and the prime
constituencies are the company's employees and suppliers and the various communities in which the
resources and the company's operations are located.
The firm has a responsibility to all these multiple stakeholders-shareholders, customers, employees,
suppliers, and communities. Ideally, its management control system should identify the goals for each of
these groups and develop scorecards to track performance.
Example: In 2005, the Acer Group, headquartered in Taiwan, was one of the largest computer
companies The Company subscribed to the multiple stakeholder approach and managed its internal
operations to satisfy the needs of several constituencies. To quote Stan 'Shih,-the founder, "The
customer is number 1, the employee is number 2, the shareholder is number 3. I keep this message
consistent with all my colleagues. I even consider the company's banks, suppliers, and others we do
business with are our stakeholders; even society is stakeholder. I do my best to run the company that
way."
Lincoln Electric Company is well known for its philosophy that employee satisfaction was more
important than shareholder value. James Lincoln wrote: "The last group to be considered is the
stockholders who own stock because they think it will be more profitable than investing more in any other
way. The absentee stockholder is not' of any value to the customer or to the worker, since he has no
knowledge of nor interest in the company other than greater dividends and advance in the price of his
stock." Donald F. Hastings, chairman and chief executive officer, emphasized that this was still the
company's philosophy in 1996.
Q.4 Explain and illustrate with one example differences between 3 forms of internal auditFinancial, Operational & Management.
Financial AuditFinancial Audit is a historically oriented, independent evaluation performed by
internal auditor or external auditor for the purpose of attesting to the fairness, accuracy and reliability of
the financial data, providing protection for the entity's assets; evaluating the adequacy and accomplishment
of the system (internal control) designed,
provide for the aforementioned Fairness and Protection, Financial data, while not being the only source of
evidence, are the primary evidential source. The evaluation is
performed on a planned basis rather than a request".
Institute of Internal Auditor:Financial audit takes care of the protective aspect of the business and it does not
normally carry out constructive appraisal function of the business operations. It helps in detection and
prevention of fraud. It also verifies whether documentation and flow of activities arc in conformity with
the internal control system introduced and developed within the organization. It helps coordinating with
statutory auditor to help them in proper discharge of their function. Besides, financial audit also ensures
compliance with statutory laws especially in financial and accounting matters.
Objectives of Financial Audit:
-To see that established accounting systems and procedures have been complied with
-To see that proper records have been maintained for the fixed assets of the Concern to look into
correctness of the financial data and records along with correctness of the accounting procedure followed.
71

-To see whether scrap, salvage and surplus materials have been properly accounted for etc.
-To see that internal control system has been working properly.
-To see that any abrupt variation in sales, purchases etc.; with respect to immediate previous year are not
due to any irregularity
-To see that the credit control has been strictly followed.
-To see that all payments have been made with proper authorization and approval. .
-To see that preparation of salary and wage pay roll has been properly done.
budgetary control system, if any scope and performance of internal audit, if any, suggestions for
improvements in performance, if any, and improved inventory policies.
The opinion expressed by the auditors shall be based on verified data, reference to ich shall also be made
here and, if practicable, included after the company has been
forded on opportunity to comment on them.
Management Audit
It is a complex task closely related with the process of management. It is highly result oriented. It requires
inter/multi-disciplinary approach as it involves examination, review and appraisal of various policies and
actions of management on the basis of certain norms/standards.
It undertakes comprehensive and critical review of all organizational activities with wider perspective.
It goes beyond conventional audit and audits the efficacy of the management itself.
Definition:
It's a comprehensive and constructive examination of an organization, the structure of a company,
institution or branch of government or of any components thereof, such as division or department and its
plans, objectives, its means of operations and its use of human and physical facilities.
William P. Leonard
It's an investigation of a business from the higher level downwards in order to ascertain whether sound
management prevails throughout, thus facilitating the most effective relationship with the outside
world and the most efficient and smooth running internally.
Leslie Howard
It is an audit performed with the object of examining the efficacy of the institution/control systems,
management procedures towards the achievement of enterprise goals.
Churchill & Cyert
It is an objective and independent appraisal of the effectiveness of managers and the effectiveness of
the corporate structure in the achievement of company objectives and policies. Its aim is to identify
existing and potential management weaknesses within an organization and to recommend ways to
rectify these weaknesses.
Chartered Institute of Management Accountants London
Thus it can be seen that management audit is an examination, review and appraisal of the various
policies and actions of the management. It is a tool for the evaluation of methods and performance in
all the areas of the enterprise.
Objectives:
1. To ascertain the provision of proper control at different levels, their effectiveness I in accomplishing
management goals.
2. Ascertain objectives of the organization are properly communicated and understood at all levels.
3. To reveal defects or irregularities in any of the elements examined and to indicate what improvements
are possible to obtain the best results of the operations of the company.
72

4. To assist the management to achieve the most efficient administration of its operations.
5. To suggest to the management the ways and means to achieve the objectives if the management of the
organization itself lacks the knowledge of efficient management.
6. It aims to achieve the efficiency of management and assess the strength and weaknesses of the
organization structure, its management team and its corporate culture.
7. To ascertain the provision of proper control at different levels, their effectiveness in
accomplishing management goals.
8. Ascertain objectives of the organization are properly communicated and understood at all levels.
9. To reveal defects or irregularities in any of the elements examined and to indicate what improvements
are possible to obtain the best results of the operations of the company.
10. To assist the management to achieve the most efficient administration of its operations.
11. To suggest to the management the ways and means to achieve the objectives if the
management of the organization itself lacks the knowledge of efficient management.
12. It aims to achieve the efficiency of management and assess the strength and weaknesses of the
organization structure, its management team and its corporate culture.
13. To help the management at all levels in the effective and efficient discharge of their duties and
responsibilities.
The auditor must apprise managerial performance at all levels of the organization. The audit starts
right at the top level of the management. It studies the managerial performance at all the levels of
management. The audit has to study the decision-making system of the organization and also the level of
autonomy granted to the managers at different levels of the organization. The authority and responsibility
given at the different levels of the management. One of the most important things that the audit must study
is that the mangers at various levels use the authority.
Conducting Management Audit
Management audit requires an interdisciplinary approach since it involves a review of all aspects of
management functions. It has to be conducted by a team of experts because this requires 3 varieties of
skills, which one individual may not possess.
The team may consist of management experts, accountants, and the operation research specialists, the
industry experts and even social scientists.
The auditors must have analytical mind and ability to look at a management function form the point of
view of the organization as a whole. They therefore have to be properly trained in this aspect. They need to
have through knowledge of the management science and they should be acquainted with the salient
features of various functional areas.
Under financial audit, the entire emphasis is on macro-aspect, the individual transactions beingscrutinized for check of the aggregates. It is concerned with examination of transactions recorded in the
books of account. It reviews the procedure and internal checks, and scrutinizes individual transactions for
the purpose of verification, of Profit and Loss Account and Balance Sheet. Financial audit is not concerned
with ~ avoidance of profiteering motive. It indicates the financial position and over~ performance of the
business, regardless of its performance in various segments. Financial audit is applicable to all classes of
companies and industries irrespective of size and Dan of operations.
Instead of serving the interest of the management and the Government, it serves interest of
shareholders. Financial audit is organization - oriented. It is conducted under Sections 224 - 232 of the
73

Companies Act 1956.


Financial Audit

Management Audit
It is concerned with the review of the past
Performance to ascertain whether it is in tune with
the objectives, policies and procedures of the
enterprise.

It is concerned with financial aspects of


business transactions of the year under
audit
The auditor examines the past financial
records to report his opinion on the truth
and fairness of the representations made in the
financial statements. Examination
of the
performance of the management is
beyond his scope

The management auditor reports on performance of


the management during a particular period and
suggest ways to remedy the deficiencies, including
modification of objectives, policies etc.

Past year '(Financial) transactions are


No limit as to the period to be covered
Covered Enterprises such as companies, trust and
societies etc.
There is legal compulsion as regards management
Financial audit is compulsory in the case of certain audit.
enterprises such as companies, trust and societies etc.
The auditor reports to the owner, i.e.
The auditor reports to the management
shareholders in the Case of a company

Q.5 Explain briefly various stages of management control process citing salient features of each.

Management control process involves communication of information to the managers at various levels of
hierarchy and their interactions arising out of them. These communications aim towards attaining the
organization's goals. But individual managers have their personal goals also. For example, a young
manager with good education, experience, personality and social background joins a company like
Britannia Industries or Reliance. The company finds him fit for the position as per job specifications,
appoints him and makes him aware of what the company expects of him. The young manager sets his
goals of gaining rich experience for his career progress besides adequate compensation packages.
Naturally, his actions will be directed towards achieving his own objectives and goals while serving the
company. Thus, his self-interest and the best interest of the organization are apparently in conflict. But the
best results can be achieved by perfectly matching the two interests and this is called 'goal congruence'.
It is quite apparent that perfect congruence between the goals of the individual and the organization
individual's goals and the organization's goals can never happen. Yet, the main purpose of a management
control system is to assure goal congruence between the interest of the individual and the organization as
far as practicable.
Management control systems
Formal and Informal Communication
As mentioned earlier, all the communication of information may be either formal or informal. The formal
communication system involves strategic plan, budgets, standards and reports whereas the informal
communication is made through letters and memos, verbally or even by facial expression.
Formal communications are all documented and addressed to the responsible managers for their
information and actions, if necessary. However, the actions depend on the perception of the individual
managers.
74

Informal communication, on the other hand, relates to some external factors-work ethics, management
style and culture. Added to these factors is the existence of an informal organization within the structured
formal organization.
Informality refers to the relaxation of sharp differentiation and explicit description of behavior as
indicated in the hierarchy and thereby, moving away from superior/subordinate relationship. However,
such relations depend on the personal capabilities of the manager such as education, experience, expertise,
trust and cooperation. For example, Accounts Manager of Nasik Plant (see the organization chart in the
diagram 3.2) reports to the General Manager of the Plant. While visiting the Corporate Office for attending
a Training Course, he meets other colleagues, parallel officers and even the Finance Director. The latter
communicates some important matter to him verbally and wants action thereon. Accounts Manager carried
out the instructions so given. As per the organization chart, he should inform his General Manager, but it
depends on his own perception of the situation, and he mayor may not report to the General Manager.
Work Ethics, Management Style and Culture
External factors like work ethics vary from place to place. Therefore, organization work culture depends
on the general behavior of the people in the society where the organization situates. Work culture
generally differs because of the life style and the attitude towards the work. For example, people of
Mumbai lead very fast life. Time has more value at Mumbai as compared to Kolkata, where people take
things easily and leisurely. Japanese and Korean people have reputation for their excellent work culture.
However, the most important internal factor is the organization's culture and climate. The culture refers to
the set of common beliefs, attitudes, norms, relationships and assumptions that are explicitly or implicitly
accepted and evidenced throughout the organization. The writer joined Union Carbide as an Assistant just
three days before Christmas Eve. On the very second day, when he attended Christmas lunch, his table
was shared by none other than the General Sales Manager Dr. W.R. Correa. He kept us amused with
various stories of his recent tour abroad and recited Urdu 'shairies', even sharing jokes. Such a situation
was unthinkable in Jessop & Co., where sharp differences were maintained at every level of hierarchy.
Management control systems
Climate is used to designate the quality of the internal environment that conditions the quality of
cooperation, the development of individuals, the extent of members' dedication or commitment to
organizational purpose and the efficiency with which that purpose is translated into results. Climate is the
atmosphere in which individuals work help, judge, and reward, constrain and find out about each other. It
influences moral-the attitude of the individual towards his/her work and environment.
Culture differs between the organizations, but cultural norms are extremely important. They are not
written like formal communication. But the existence of a good culture can be felt from the behavior of the
members of the organization. Once the writer landed up with his family at Hyderabad in the early morning
to discover that nobody had come to receive them at the station. His visit was arranged through non other
than the Director of the company himself. His unit being new, telephone directory did not include any
number of his unit, but the parent organization's telephone number was located. When an executive of the
parent company was contacted, he immediately sent an officer of the company with a car to pick us up to
their Guest House, entertain with coffee and then put up in a Hotel. What subsequently happened is a
different matter, but the attitude and treatment of that member of organization speak volumes about their
excellent culture.
In any organization, the culture remains unchanged as long as the Chief Executive remains in position.
When a new executive replaces him, there is likelihood of some change in the culture, unless the new
Chief follows the footsteps of his predecessor and maintains it. Generally, if higher positions are filled in
through promotion of internal executives, the culture remains unchanged and the traditions are maintained.
The other important internal factor which influences management control system is management stylethat is the attitude of the superior to his subordinates and the latter's reaction through their perception of
the attitude of their superiors. Again, the attitude ultimately stems from the temperament of the Chief
Executive, who controls the entire organization. That is why R. W. Emerson said "an institute is the
lengthened shadow of a man".

75

Importance of Informal Communication


An organization indulges in informal control process when encountering non-routine decision-making or
when seeking new information to increase understanding of some problem areas. During a very critical
period in an organization, the writer found that the Chief Executive used to call managers informally at his
residence or club to extract information in a relaxed manner rather than in a tense situation prevailing in
the factory.
Formal Control Process
Formal communication system is structured as per the 'hierarchy outlined in the organization chart. The
system has the following four components:
(a) Strategic plan and programme
(b) Budgeting
(c) Operations and measurement in responsibility centers (d) Reporting

(a) Strategic Plan and Programme


The foundation of management control process lies in the organization's goals and its strategies for
attaining these goals. A strategic plan is prepared in order to implement the strategies, after carefully
considering opportunities and threats in the external environment as well as the strengths and weaknesses
in the internal environment. Thus, a strategic plan and programme is prepared as a guideline to budgeting.
(b) Budgeting
The strategic plan is converted to an annual budget incorporating planned expenditure and revenues for
individual responsibility centers. Expenses and revenues are marked for each responsibility centre period
wise, say monthly, quarterly, half yearly, and annually.
(c) Operations and Measurement
Responsibility centers operate within the framework of the budget, established standards, standing
instructions, practices and operating procedures embodied in 'rules', and 'manuals'. Thus, besides budget,
the responsibility centers are also guided by a large number of rules. They record the resources actually
used and revenue earned. They also classify the data by programmes as well as by responsibility centers
for performance measurement.
(d) Reporting
Actual performance is analyzed, measured and reported against plan, indicating variances and highlighting
areas of weaknesses. If the performance is satisfactory, feedback information is sent to the responsibility
centre concerned for praise or reward. If the same is unsatisfactory feedback communication is sent to the
responsibility centre concerned for corrective action. If such action requires to be included in the budget,
then the latter is revised to give effect to the changed position. If required, then the plan itself can be
revised and a new basis of control may be established.
The aforesaid formal control process has been presented in the following diagram:

76

77

You might also like